You are on page 1of 43

INDEX

BASIC SURGERY 1
TRAUMA AND SHOCK 14
BREAST SURGERY 18
ENDOCRINE 24
VASCULAR SURGERY 30
PLASTIC SURGERY 39

Let the young know they will never find a more interesting, more
instructive book than the patient himself.

—Giorgio Baglivi
BASIC SURGERY 10- Regarding total body water all the following are true
EXCEPT:
1- Severe sepsis is differentiated from sepsis by: a- Females and obese persons have an increased
percentage of body water.
(A) A history of premorbid condition such as diabetes b- Increased muscle mass is associated with increased
(B) Positive blood cultures for bacteria or fungus total body water.
(C) Acute organ failure such as renal insufficiency c- Newborn infants have the greatest proportion of total
(D) Prolonged arterial hypotension body water.
d- Total body water decreases steadily with age.
2- Which of the following is the most effective dosing
of antibiotics in a patient undergoing elective colon 11- Regarding extracellular fluid all the following are true
resection? EXCEPT:
(A) A single dose given within 30 min prior to skin incision a- The total extracellular fluid volume represents 20% of
(B) A single dose given at the time of skin incision the body weight.
(C) A single preoperative dose + 24 hours of postoperative b- The plasma volume constitutes 5% of body weight.
antibiotics c- Potassium is the principal cation in extracellular fluid.
(D) A single preoperative dose + 48 hours of postoperative d- The interstitial fluid equilibrates rapidly with the other
antibiotics body compartments.
3- The antibiotic of choice in a penicillin allergic patient 12- Total body water constitutes what percentage of
undergoing a cholecystectomy for acute cholecystitis is: adult body weight:
(A) Ertepenem (B) Ceftriaxone a. 40 b. 50
(C) Vancomycin + Metronidazole c. 60 d. 70
(D) Flouroquinolone + Metronidazole
13- Insensible loss of fluid from skin and lungs for 24
4- Appropriate duration of antibiotic therapy for most hours is normally in the span of:
patients with bacterial peritonitis from perforated
appendicitis is: a. 100 – 250 ml. b. 250 – 500 ml.
c. 500 – 750 ml. d. 750 – 1000 ml.
(A) 3-5 days (B) 7-10 days
(C) 14-21 days (D) > 21 days 14- Which of the following diagnoses would be most
likely in a patient who presents with normovolemic
5- Which of the following is NOT a risk factor for hyponatremia ?
developing a surgical site infection (SSI)?
(A) SIADH (B) High output renal failure
(A) Radiation exposure (B) Recent surgery (C) Water toxicity (D) GI losses
(C) Prolonged hospitalization (D) Infancy
15- Which of the following can contribute to
6- Which of the following is most suggestive of a hyperkalemia in patients with renal insufficiency?
necrotizing soft tissue infection and would mandate
immediate surgical exploration? (A) Loop diuretics (B) Aspirin
(C) Calcium channel blockers (D) NSAIDs
(A) A small amount of grayish, cloudy fluid from a wound
(B) Red, swollen extremity which is tender to palpation 16- Which of the following would cause dereased deep
(C) Soft tissue infection with a fever > 400C tendon reflexes ?
(D) Induration with pitting edema on the trunk
(A) Hypokalemia (B) Hypomagnesemia
7- The appropriate duration of antibiotic therapy for (C) Hpocalcemia (D) Hypoglycemia
nosocomial urinary tract infection is:
17- Which of the following is an early ECG change seen in
(A) 3-5 days (B) 7-10 days hyperkalemia ?
(C) 21 days
(D) Until the patient is asymptomatic and the urinalysis is (A) Prolonged PR interval (B) Sine wave formation
normal (C) Peaked T waves (D) Flattened P wave
8- The metabolic response to a major operation or severe 18- A postoperative patient with potassium of 2.9 is given
injury includes all the following EXCEPT: 1 mEq\kg replacement with KCL. Repeat tests after the
replacement show the serum K to be 3.0 The most likely
a- Hypermetabolism. b- Fever. diagnosis is:
c- Decreased gluconeogenesis. d- Tachypnea.
(A) Hypomagnesemia (B) Hypocalcemia
9- Causes of persistence of a sinus include all of the (C) Metabolic acidosis (D) Metabolic alkalosis
following EXCEPT:

a. Specifie chronic infection e.g. TB.


b. Foreign body.
c. Inadequate drainage of the cavity
d. Prolonged use of antibiotics.

1 EDITED BY: MOHAMED ELSHEBL


19- Which of the following is a cause of acute a. 7.05 -7.19. b. 7.20-7.35.
hypophosphatemia? c. 7.36-7.44. d. 7.45 - 7.59.

(A) Chronic ingestion of magnesium containing laxatives 30- Which of the following contributes maximum to
(B) Insulin coma body fluid osmolality:
(C) Refeeding syndrome
(D) Rhabdomyolosis a. Glucose. b. Sodium.
c.Chloride. d. Bicarbonate.
20- Which of the following should be the first treatment
administrated to a patient with potassium level of 6.3 31- The standard bicarbonate level in the plasma is
and flattened P wave on his ECG? normally:

(A) Kayexalate (B) Insulin and glucose a. 12 - 15 mmol/l. b. 16 - 21 mmol/1.


(C) Calcium gluconate (D) Inhaled albuterol c. 22 - 25 mmol/1. d. 26 - 31 mmol/l.

21- Insensible water loss from lungs is: 32- Concerning metabolic acidosis, which of the
following statements is untrue:
a. 200 ml. b. 500 ml.
c. 1000 ml. d. 1500 ml. a. Occurs in diabetes, starvation, shock and anuria.
b. May be due to diarrhoea and intestinal fistulas.
22- The extracellular fluid differs from the intracellular c. Is often associated with air hunger.
fluid in all of the following EXCEPT: d. Causes bradycardia and hypotension.

a. Protein content. b. Electrolyte composition. 33- The ideal infusion fluid for correction of
c. Volume. d. Osmolarity (tonicity). hypokalaemic alkalosis due to pyloric obstruction is:

23- Concerning the sodium ion, which statement is a. Normal saline.


untrue among the following? b. Potassium chloride in 5% glucose.
c. Ringer's solution. d. Ammonium chloride.
a. Diffuses readily through cell membranes.
b. Is the major cation of the extracellular fluid. 34- All of the following components or qualities of stored
c. Is the chief regulator of body water. whole blood tend to decrease over time EXCEPT:
d. Its urinary excretion is reduced after trauma.
a- Red blood cell survival time.
24- Among the following statements about potassium, b- Potassium concentration.
which one is untrue? c- Platelet activity.
d- Oxygen-carrying capacity.
a. Is the major cation of the intracellular fluid.
b. Its intracellular concentration is about 150 mEqlL. 35- Allowing blood which is ready for transfusion to
c. Its average daily need is about 60 mEqlL. remain for four hours in a warm environment:
d. Is well-conserved by the kidney after trauma.
a. Reduces shock. b. Increases shock.
25- Potassium deficiency is present, if the plasma- c. Favors subsequent hepatitis.
potassium level is less than: d. Encourages bacterial proliferation and septicaemia.

a. 5 mmolll b. 4.5 mmol/l. 36- The peak number of fibroblasts in a healing wound
c. 4 mmolll. d. 3.5 mmol/l. occurs:

26- Concerning potassium depletion, the incorrect (A) 2 days after injury (B) 6 days after injury
statement among the following is: (C) 15 days after injury (D) 60 days after injury

a. Follows loss of gastrointestinal secretions. 37- Macrophages are present in the wound starting on
b. May be produced by diuretics. the 4th day after injury until the wound is completely
c. Is often associated with acidosis. healed. The primary function of macrophages in wound
d. Predisposes to cardiac arrhythmias. healing is:

27- Symptoms of hyperkalaemia is reflected in which (A) Intracellular killing of bacteria


organ system: (B) Collagen production
(C) Activation of cell proliferation
a. Respiratory. b. Gastrointestinal. (D) Modulation of the wound environment
c. CVS. d. CNS.
38- The first cells to migrate into a wound are :
28- Hypokalaemia causes:
(A) Macrophages (B) T lymphocytes
a. Peaked P waves. b. Adynamic ileus. (C) PMNs (D) Fibrblasts
c. Hyper-reflexia. d. All ofthe above.
39- There are 18 types of collagen in the human body.
29- In health the pH of the blood lies between the Which two are the most important in wound healing?
ranges:
(A) Type I and III (B) Type III and VIII
(C) Type II and X (D) Type VI and XII

SURGERY Q-BANK VOL. 1 2


40- The tensile strength of a completely healed wound 50- Regarding enteral route of nutrition all the following
approaches the strength of uninjured tissue: are true EXCEPT:

(A) 2 weeks after injury (B) 3 months after injury a. May cause bacterial overgrowth in the gut.
(C) 12 months9 after injury (D) Never b. Reduces stress ulceration.
c. Is less expensive than the parenteral route.
41- How does re-epithilization take in a well- d. Decreases the incidence of cholestasis by promoting
approximated surgical wound ? biliary flow.

(A) 2 days (B) 1 week 51- The following are known to cause a metabolic
(C) 2 weeks (D) 1 month acidosis EXCEPT

42- Which phase of healing is most affected by a. ischaemic bowel b. diabetic ketoacidosis
exogenous corticosteroids ? c. sepsis d. persistent vomiting

(A) Initial phase of cell migration and angiogenesis 52- All of the following statements regarding the anion
(B) Proliferative phase gap are true EXCEPT
(C) Maturation (D) Scar remodeling
a. normal range is between 10 and 19 mmollL
43- Which of the following should be given to promote b. it reflects the concentrations of normally unmeasured
wound healing in patients receiving corticosteroids? anions in the serum
c. It is increased in diabetic ketoacidosis
(A) Vitamin A (B) Vitamin B1 d. It is increased in chronic diarrhea.
(C) Vitamin B2 (D) Vitamin C
53- The following are absorbable sutures EXCEPT:
44- Which of the following should be performed in a
patient with a suspected Marjoline ulcer? a. Catgut. b. Polypropylene (prolene).
c. Polydioxanone (PDS). d. Polyglactin (Vicryl).
(A) Hyperbaric therapy for 6 weeks
(B) Zinc supplementation 54- Regarding crystalloid and colloid solutions
(C) Oral tetracyclin for 6 weeks
(D) Biopsy a. Normal saline contains 154 mmol sodium and 154
mmol of chloride.
45- Which of the following is considered the most b. Hartmann's solution contains calcium and bicarbonate.,
effective therapy for venous stasis ulcer? c. Albumin has a half life in the circulation of about 15
hours.
(A) Supplemental vitamin A d. Dextrans reduce platelet aggregation and can induce
(B) Topical antibiotic ointment anaphylaxis he daily maintenance.
(C) Compression therapy
(D) Hyperbaric therapy 55- Regarding acute respiratory distress syndrome all
are true EXCEPT:
46- Which of the following is most likely to cause a
diabetic ulcer? a. Caused by direct or indirect lung injury e.g. sepsis,
trauma.
(A) Uncontrolled hyperglycemia b. The main pathological feature is diffuse alveolar
(B) Large vessel ischemia damage
(C) Small vessel ischemia (D) Neuropathy c. Bilateral infiltrates present on chest x-ray.
d. Hypoxaemia is often respond easily to respiratory
47- A teenage African American girl presents with support.
large keloids on both earlobes 12 months following ear
piercing. Which therapy should be added to surgical 56- Which of the following is the best initial treatment of
debulking of the lesions? a burn with hydrofluoric acid?

(A) None – surgical resection alone is sufficient as the (A) Copious irrigation with water
initial therapy (B) Copious irrigation with a dilute solution of sodium
(B) Intralesional corticosteroids bicarbonate
(C) Pressure earrings (D) Radiation therapy (C) Application of a topical quaternary ammonium
compound
48- An alginate dressing is best used in which of the (D) Application of topical calcium carbonate gel
following wounds?
57- Staphylococcal scalded skin syndrome is most likely
(A) An open traumatic wound to be associated with wgich of the following?
(B) An open surgical wound
(C) An infected wound (A) Phenytoin (B) Barbiturates
(D) A partial thickness burn wound (C) Tetracycline (D) Otitis media

49- Coumadin effect can be reversed by: 58- Which of the following is the most form of basal cell
carcinoma?
a. Vit. C. b. Calcium.
c. Vit K1 d. Vit K2 (A) Morpheoform (B) Superficial spreading
(C) Pigmented (D) Nodular

3 EDITED BY: MOHAMED ELSHEBL


59- A Marjolin›s ulcer arises in areas exposed to : 67- The MOST common cause of empyema is:

(A) External beam radiation (B) Thremal injury a- Pneumonic process in the underlying lung.
(C) Pressure (D) Lymphedema b- Rupture of an emphysematous bulla.
c- Penetrating injury of the chest.
60- Angiosarcoma associated with Steward-Treves d- Subphrenic abscess.
syndrome arises in areas exposed to:
68- Shock can best be defined as:
(A) External beam radiation (B) Thremal injury
(C) Pressure (D) Lymphedema a- Hypotension. b- Hypoperfusion of tissues.
c- Hypoxemia. d- All of the above.
61- Which type of melanoma has the best overall
prognosis? 69- In haemorrhagic shock all of the following are
present EXCEPT:
(A) Superficial spreading (B) Nodular
(C) Lentigo maligna (D)Acral lentiginous a- Low cardiac output. b- Decreased venous return.
c- Peripheral pooling. d-Increased peripheral resistance.
62- A 75-year-old farmer complained of a scaly,
plaque like skin lesion on his forearm with recent 70- Class 4 hypovolemic shock is present in presence of
development of ulceration. Biopsy reveals invasive all EXCEPT:
squamous carcinomas within actinic keratosis negative
examination of axillary nodes. Definitive treatment is: a- Blood loss more than 40% b- Heart rate> 140 min.
c- Urine output 10-15 ml/hr. d- Drowsy, confused patient.
(A) Local wound care until the ulcer heals; then wide
excision and repair 71- SIRS may occur as a result of:
(B) Excision of the lesion with frozen section determined
free margins and repair a- Infection: bacterial, viral, fungal b- Trauma & Burns
(C) Wide excision; split-thickness skin graft and axillary c- Pancreatitis d- All of the above
node dissection
(D) Wide excision; split-thickness graft and radiation 72- Septic shock responds best to:
therapy
a- Massive antibiotics. b- Adrenocortical steroids.
63- A43-year-old window cleaner fell off a scaffold. He c- Drainage of septic collections. d- Vasopressors.
sustained an open wound on the right leg. Debridement
was carried out in the emergency department, and 73- Banked blood is deficient in all of the following
the edges of the wound were left open. The wound EXCEPT:
measures 4 cm × 6 cm. What is TRUE of wound
contraction? a- Platelets. b- White cells.
c- Oxygen-carrying, capacity of Hb. d- Potassium.
(A) It occurs within 12 hours of injury.
(B) It is more prominent over the tibia than gluteal region. 74- Platelets survive in stored blood for:
(C) It is accelerated if wound is excised3 days after injury.
(D) It is experimentally less affected by excision of tissue a- 1 day. b- 2 days.
from center of wound rather than at the periphery. c- 5 days. d- 7 days.

64- Which factor is least likely to inhibit wound 75- The most serious complication of blood transfusion
contraction? is:

(A) Radiation (B) Cytolytic drug a- Pyrogenic reactions.


(C) Transformation growth factor b b- Thrombophlebitis of recipient vein.
(D) Full-thickness skin graft c- Circulatory overloading. d-Incompatibility reactions.

65- A29-year-old female swimmer develops a pigmented 76- The most important clue to mismatched transfusion
lesion on the right thigh. With reference to a pigmented in an anaesthetized patient is:
lesion, there is an increased risk of developing
melanoma if it is identified with which of the following? a- ECG changes. b- Hypotension.
c- Bleeding. d- Tachycardia.
(A) Hutchinson freckle (lentigo maligna)
(B) Freckle involving basal layer of skin 77- Common indications for nutritional support include:
(C) Congenital nevocellular nevi (D) Tophi
a- Ileus more than 4 days. b- Massive bowel resection.
66- A 38-year-old female undergoes removal of a2 × 1-cm c- Intestinal fistula. d- All of the above.
skin lesion shown to be a melanoma. It is reported as
Clark level 1, which implies what? 78- Complications of TPN include:

(A) It is superficial to the basement membrane. a- Sepsis. b- Pneumothorax


(B) It is 1 mm in thickness. c- Hyperosmolar coma. d- All of the above.
(C) It has nodal involvement.
(D) It involves the papillary layer.

SURGERY Q-BANK VOL. 1 4


79- Regarding the role of the gut in shock and sepsis all 86- Regarding Warfarin all are true EXCEPT:
are true EXCEPT:
a- Reduces the concentration of vitamin A dependent
a- Selective decontamination of the digestive tract with clotting factors.
the use of oral antibiotics has been shown to reduce b- Has a half life of about 36 hours.
nosocomial pneumonias and to improve mortality rates. c- Crosses the placenta and should be avoided in
b- Enteral nutrition preserves the villus architecture of the pregnancy.
gut. d- Doses should be reduced in liver disease.
c- Gut dysfunction may contribute to the development of
MODS by bacterial translocation. 87- Heparin:
d- As compared with parenteral nutrition, enteral nutrition
is associated with a reduction in septic morbidity. a- Has a half life of 90 minutes.
b- Can be reversed by protamine sulphate.
80- Blood group mismatching can be accepted for c- Can induce thrombocytopenia.
which transplant: d- All are true.

a- Kidney. b- Liver. 88- Regarding Disseminated intravascular coagulation


c- Heart. d- None of the above. (DIC) all the following are true EXCEPT:

81- Concering immunology in liver transplantation all are a- Characterized by simultaneous activation of
true EXCEPT: coagulation and fibrinolytic pathways.
b- Present by bleeding, but thrombosis may occur.
a- Good human leukocyte antigen (HLA) matching c- Is associated with sepsis, trauma, malignancies and
between recipient and donor is mandatory for a good obstetric emergencies.
outcome for liver transplantation. d- Is associated with decreased fibrin degradation
b- Hyperacute rejection is almost nonexistent following products (FDP).
liver transplantation.
c- Acute rejection occurs in 30- 50% of patients and is 89- The most important aspect in treating disseminated
reversible in most patients with large doses of steroids. intravascular coagulation (DIC) is to:
d- Chronic rejection, is usually irreversible, and often
requires retransplantation. a- Administer heparin. b- Administer platelets.
c- Treat the underlying disease process.
82- Regarding The prothrombin time all the following d- Achieve normal levels of fibrinogen.
are true EXCEPT:
90- The most frequent hereditary bleeding disorder is:
a- Measures the activity of the extrinsic coagulation
pathway. A. Haemophilia A. B. Haemopjilia B.
b- Is not usually prolonged in liver disease. C. Von Willebrand disease.
c- Is normal in haemophilia A. D. Immunogenic thrombocytopenic purpura.
d- Can be expressed as the INR when monitoring warfarin
dosage. 91- A patient receiving warfarin therapy presents for
parotid tumour surgery and his laboratory finding
83- Regarding low-molecular-weight heparins choose showed INR of 2.3. What would be the appropriate
the correct answer: treatment?

a- Have a longer half-life than unfractionated heparins A. Stop warfarin and replace it with SC heparin for 48
b- Act predominantly on factor Xa. . hours tilllNR is below 1.4 and then proceed with surgery.
c- Inspite of adequate anticoagulation APTT remains B. Proceed immediately with surgery without stopping
within the normal limits d- All are true. warfarin.
C. Stop warfarin, give fresh frozen plasma (FFP) and
84- Regarding Fresh-frozen plasma all the following are proceed to surgery
true EXCEPT: D. Stop warfarin and proceed with surgery in 24-48 hours.

a. Contains all non-cellular components of blood 92- Pharyngeal pouch should be suspected in presence
including all clotting factors, immunoglobulin and plasma of:
proteins
b. Cryoprecipitate contains Icss fibrinogen than FFP. a- Dysphagia. b- Regurgitation of undigested food.
c. Stored at- 30°C for up to 1 year. c- Aspiration pneumonitis. d- All of the above
d. FFP contain anti-A and anti-B antibodies which may
cause hemolytic reaction . 93- Regarding a pharyngeal pouch which statement is
incorrect:
85- In assessment of bleeding disorders choose the
correct answer. a- It protrudes through Killian›s dehiscence.
b- It usually turns to the left side of the neck.
a. Normal APTT and PT indicate platelet or vessel defect. c- It may be visible in the neck.
b. Abnormal APTT and PT indicate defect in common d- It is twice as common in males as in females.
pathway
c. Normal APTT and abnormal PT indicate defect of 94- Branchial cyst commonly arises from remnant of:
extrinsic pathway d. All are true.
a- First branchial cleft. b- Second branchial cleft.
c- Third branchial cleft. d- All of the above

5 EDITED BY: MOHAMED ELSHEBL


95- Regarding branchial cyst which statement is a- Accessory nerve. b- Internal jugular vein.
incorrect? c- Carotid artery. d- Submandilular gland.

a- Arises from the second branchial cleft. 107- Potato tumor is a:


b- Usually appears between the ages of 20 – 25 years.
c- Protrudes beneath the anterior border of the a- Carotid body tumor. b- Sternomastoid tumor.
sternomastoid. c- Cystic hygroma. d- Branchial cyst.
d- All are correct.
108- Cystic hygroma is a:
96- Branchial cyst is best differentiated from cold a Haemangioma. b- Sebaceous gland tumor.
abscess by: c- Meningioma. d- Lymphangioma.

a- Fluctuation. b- Transillumination 109- Which of the following is brilliantly translucent:


c- Contains cholesterol crystals d- None of the above
a- Sebaceous cyst. b- Dermoid cyst.
97- Characteristics of cystic hygroma include all EXCEPT: c- Cystic hygroma. d- Branchial cyst.

a- Develops from jugular lymph sacs. 110- In Hodgkin›s lymphoma, spot the wrong statement:
b- Brilliantly translucent.
c- Typically occupies the middle third of neck. a. It has a better prognosis than NHL.
d- Enlarges when the child cries. b. It is 21haracterized by presence of few malignant cells
outnumbered by inflammatory cells.
98- Which of the following is inappropriate to cystic c. It is 21haracterized by presence of Reed – Sternberg
hygroma: cells.
d. Diagnostic laparotomy is essential in all patients.
a- It is a type of cavemous haemangioma.
b- It can be the earliest SWelling of the neck to appear in 111- Most common site of enlargement of lymph nodes in
life. Hodgkin›s lymphoma is:
c- It can obstruct labour. d- It is brilliantly translucent.
a- Abdominal. b- Cervical.
99- Treatment strategy in Ludwig angina includes: c- Axillary. d- Mediastinal.

a- Amoxycillin plus metronidazole. 112- Most common presentation of Hodgkin's lymphoma


b- Decompression of both submandibular triangles. is:
c- Tracheostomy. d- All of the above.
a- Leukocytosis. b- Fever.
100- A ranula is a: c- Painless enlargement of lymph nodes. d- Pruritus.

a- Cystic swelling in the floor of the mouth. 113- Hodgkin›s lymphoma with right sided neck nodes
b- Forked uvula. and left inguinal node without fever is of:
c- Sublingual thyroid. d- Thyroglossal cyst.
a- Stage Ia b- Stagc IIIa
101- The ‹potato› tumour of the neck is a: c- Stage IIa d- Stage IVa.

a- Sternomastoid tumour. b- Carotid body tumour. 114- syptoms of Hodgkin›s lymphoma include all the
c- Thyroid tumour. d- Parotid tumour. following EXCEPT:

102- Laryngocele is common to: a- Pruitus. b- Headache.


c- Weight loss. d- Fever.
A. Trumpet blowers. B. Glass blowers.
C. Patients of chronic cough. D. All of the above. 115- Worst prognosis in Hodgkin›s lymphoma is:

103- Tubercular cervical lymphadenitis commonly a- Lymphocytic depletion. b- Mixed cellularity.


affects: c- Lymphocyte predominance. d- Nodular sclerosis.

A. Posterior triangle nodes. 116- About anatomy of the submandibular salivary gland,
B.Upper deep cervical nodes. all the following statements are true, except:
C.Lower deep cervical nodes. D.Submaxillary nodes.
A. The mylohyoid muscle divides the gland into superficial
104- Cervical rib may present by which of the followings and deep parts.
B. The submandibular duct arises from the superficial
a- Numbness in fingers. b- Bruit around clavicle. part of the gland.
c- Lump in lower neck. d- All of the above. C. The deep part of the gland is related to the hypoglossal
nerve.
105- Pain in the arm due to cervical rib is caused by: D. The submandibular duct is closely related to the lingual
nerve.
a- Compression of T1 b- Compression of C7.
c- Muscle ischemia. d- All of the above.

106- In radical neck dissection all the following


structures are removed EXCEPT:

SURGERY Q-BANK VOL. 1 6


117- The commonest discrete lump of the parotid gland A. A branchial cyst is observed in the neonate.
is: B. A branchial fistula is present at birth.
C. A branchial cyst is partially covered by the
A. Pleomorphic adenoma. sternomastoid muscle.
B. Mucoepidermoid carcinoma. D. The main differential diagnosis of branchial cyst is cold
C. Mumps. abscess.
D. Parotid gland stone.
126- The classic complete neck dissection for cervical
118- About submandibular salivary stones, all the lymph node metastases includes removal of all the
following statements are true, except: following, except:

A. The majority are radio-opaque. A. Excision of the anterior and posterior triangle nodes on
B. A stone can be detected by CT scan. one side.
C. A stone may present by a submandibular swelling that B. The internal jugular vein.
increases in size by eating. C. The common carotid artery.
D. A submandibular salivary swelling can be rolled over D. The sternomastoid muscle.
the angle of the mandible.
127- Do not wait for fluctuation of an abscess to drain it
119- The possible complications of submandibular in all the following positions, except:
sialadenectomy include all the following, except:
A.Distal pulp space infection of a finger.
A. Wound haematoma. B. Parotid abscess.
B. Deviation of the tongue to the operation side when C. Subcutaneous abscess of forearm.
protruded. D. Perinephric abscess.
C. Drooping of the angle of the mouth on the side of
surgery. 128- The treatment of a 2cm malignant ulcer on the left
D. Inability to close the eyelids on the side of the side of the anterior part of the tongue with no palpable
operation. neck lymph nodes is:

120- About pleomorphic adenoma of the parotid gland, A. Combination chemotherapy alone.
all the following statements are true, except: B. Neoadjuvant chemotherapy then wide surgical excision
of the tumour.
A. It has epithelial and mesenchymal components. C. Excision of the tumour with a safety margin.
B. The tumour is painless. D. Excision of the tumour with a safety margin and left
C. It has an incomplete capsule. block neck dissection.
D. The tumour is usually present deep to the facial nerve.
129- About carotid body tumour, all the following
121- About adenolymphoma, all the following statements are true, except:
statements are true, except:
A. It is a chemodectoma.
A. The tumour is related to smoking. B. It presents by a mass in the carotid triangle.
B. It may be bilateral. C. The mass is pulsatile.
C. It has epithelial and lymphoid components. D. Biopsy is needed before excision.
D. Facial nerve palsy is characteristic of this tumour.
130- About carcinoma of the lip, all the following
122- A baby is born with a left posterior triangle swelling. statements are true, except:
The swelling›s subcutaneous, partially compressible and
translucent. The clinical diagnosis is: A. The majority arise on the lower lip.
B. Its development is related to prolonged sun exposure.
A. Sternomastoid tumour. B. Branchial cyst. C. This is a squamous cell carcinoma.
C. Haemangioma. D. Cystic hygroma. D. In the absence of lymph node spread, prophylactic
neck dissection is indicated.
123- The commonest swellings in the carotid triangle are:
131- About cleft lip and palate, all the following
A. Lymphadenopathy. B. Carotid body tumour. statements are true, except:
C. Branchial cyst. D. Lipoma.
A. They are commonly associated with other congenital
124- About cold abscess of the neck, all the following anomalies.
statements are true, except: B. Cleft palate causes feeding difficulties for the baby.
C. Otitis media commonly affects babies with cleft palate.
A. It is usually caused by caseation of tuberculous D. The ideal age for cleft palate repair is 4-6 years.
lymphadenitis.
B. The abscess may have two components; one superficial, 132- A sequestration dermoid cyst is:
and one deep to the deep fascia.
C. The condition is mildly painful with low-grade fever. a. Due to squamous cells being driven in by a needle.
D. Drainage is achieved by incision. b. Due to cells being buried during development.
c. May affect limbs.
125-About branchial cyst and fistula, all the following d. A variety of sebaceous cyst.
statements are true, except:

7 EDITED BY: MOHAMED ELSHEBL


133- The wall of a True cyst can be lined by all EXCEPT: a- Much less common than squamous-cell carcinomas.
b- Very rare in oriental and black races.
a. Epithelium. b. Granulation tissue. c- Particularly common in tropical regions. .
c. Endothelium. d. All of the above. d- Characterized histologically by dark-staining solid
masses of cells arising from the basal layer of the
134- A punched out edge is a characteristic of which type epidermis.
of ulcer:
145- Cancers commonly metastasizing by blood are all
a- Tuberculous. b- Rodent ulcer. EXCEPT:
c- Syphilitic. d- Non-specific ulcer.
a- Thyroid. b- Breast.
135- A decubitus ulcer is: c- Lung. d- Brain.

a. A venous ulcer. b. An ulcer in the region of the elbow. 146- The treatment of a primary malignant melanoma of
c. A pressure sore. d. An ulcer of the tongue. the skin is:

136- The following statements about bed-sores a- Wide excision. b- Radiotherapy.


(Decubitus ulcers) are correct EXCEPT: c- Cytotoxic therapy. d- Immunotherapy.

a- Occur in bed-ridden patients. 147- Which statement is untrue concerning malignant


b- Usually involve pressure points on the back. melanoma:
c- Result from pressure ischaemia and sloughing.
d- Never penetrate deeply to involve muscle and bone. a- Is common in children and negroes.
b- Usually occurs between the ages of 50 and 60 years.
137- Which statement is incorrect concerning varicose c- May arise de novo or in a benign pigmented naevus.
ulcers: d- Always carries a bad prognosis.

a- Are always chronic and often recurrent. . 148- The prognosis of patients with malignant melanoma
b- Occur most often on the medial aspect of the lower depends on:
third of the leg.
c- Are always associated with superficial varicosities. a- Depth of invasion. b- Clinical stage ofthe disease.
d- Are often surrounded by an area of induration, c- Location of the tumour. d- All of the above.
pigmentation, oedema and dermatitis.
149- Nerve commonly affected by plexiform
138- Marjolin`s ulcer is: neurofibromatosis is:

a- Tuberculous ulcer. b- Fungal ulcer. a- Glossopharyngeal. b- Peripheral.


c- Scar carcinoma. d- Syphilitic ulcer. c- Trigeminal. d- Facial.

139- All are features of Marjolin›s ulcer EXCEPT: 150- The commonest solid tumour in infants is:

a- Slow growth. b- Pain. a- Leukaemia b- Hepatopblastoma


c- Associated with chronic scar. c- Wilm›s tumour d- Neuroblastoma.
d- Lymph node involvement is a late feature.
151- Which of the following tumors may cause elevated
140- What is not true of hamartomas: CEA levels?

a- Overgrowth of tissue at abnormal location. a- Breast cancer. b- Colorectal cancer.


b- Overgrowth of normal tissue at normal location. c- Gastric cancer. d- All of the above.
c- Virtually benign.
d- Common example is benign mole 152- Regarding chemotherapy choose the correct
answer:
141- Characteristic features of lipomas are all EXCEPT:
a. Adjuvant chemotherapy is usually given after surgery
a- Slowly growing. b- Universal tumor. where all detectable disease has been removed
c- No definite edge. d- Most are painless. b. Neoadjuvant therapy is treatment given before primary
therapy.
142- Lipoma which undergo malignant degeneration is c. The most common reason for neoadjuvant therapy is
to reduce the size of the tumor so as to facilitate more
a. Retroperitoneal b. Subserosal effective surgery.
c. subfascial d. Submucosal d. All are true.

143- Locally invasive tumors are all EXCEPT: 153- The most important factor that affects wound
healing is:
a- Basal cell cancer. b- Mixed salivary tumours.
c- Melanoma. d- Bronchial adenoma. A. Wound closure within 2 hours of injury.
B. Proper choice of suture material.
144- Concerning basal-cell carcinomas, the False C. Daily change of wound dressing.
statement is that they are: D. Adequate vascularity of the edges.

SURGERY Q-BANK VOL. 1 8


154- Regarding wound healing, the following statements A. Wound irrigation and delayed primary closure is
are all correct, except: preferred.
B. Urgent repair of cut tendons provides the best results.
A. Hypoproteinaemia impairs wound healing. C. Immediate repair of cut nerves by nerve graft is advised:
B. Immobilization of the wounded part helps wound D. Extensive debridement of muscles is necessary.
healing.
C. Corticosteroid intake impairs wound healing. 162- About healing by secondary intention, all the
D. The intake of cancer chemotherapy delays wound following statements are true, except:
healing, (All Are True)
A. Wound infection leads to healing by secondary
155- About wound management, one statement only of intention.
the following is correct: B. It is a slow process.
C. It ultimately produces a neat scar.
A. Debridement is the removal of dead tissues and D. Wound contraction is involved in this type of healing.
foreign bodies.
B. Debridement is necessary for clean incised wounds to 163- About scars, all the following are true, except:
get good drainage.
C. Nerve repair in lacerated crushed wounds is of first A. Contracture impairs joint movement.
priority. B. Squamous cell carcinoma may arise on top of a scar.
D. In fresh clean-cut wounds mark the cut tendons for C. A scar regains the original tissue tensile strength in 6
delayed treatment. months.
D. A superficial burn leaves no scar.
156- Factors that make lacerated wounds more liable to
infection include the following, except: 164- The injuries that are induced by high velocity
missiles are attributed to all the following, except:
A. Presence of foreign bodies.
B. Ischemic or dead tissues. A. Passage of the missile in its track through the tissues.
C. Suture of the wound under tension. B. Sonic waves. C. Cavitational effect.
D. Leaving the wound open after debridement. D. Neatly slicing the tissues.

157- Delayed primary closure of the skin means: 165- A keloid

A. Closure of the skin by sub-cuticular sutures. A. Is more frequently seen in black people than
B. Skin closure on the 12th day after surgery. Caucasians.
C. Skin closure when pus stops to drain from the wound. B. Never follows a puncture wound.
D. Skin closure on the 5th day. C. Stops growing after 6 months.
D. Never develops in a sternotomy incision.
158- Postoperative wound infection can produce the
following, except: 166- The most important factor that affects wound
healing is:
A. Stitch sinus. B. Reactionary hemorrhage.
C. Septicemia. D. Incisional hernia. A. Wound closure within 2 hours of injury.
B. Proper choice of suture material.
159- Incised wounds C. Daily change of wound dressing.
D. Adequate vascularity of the edges.
A. Heal faster than lacerated wounds.
B. Are more likely to injure deeper structures than missile 167- Regarding wound healing, the following statements
wounds. are all correct, except:
C. The accompanying shock waves produce extensive local
damage. A. Hypoproteinaemia impairs wound healing.
D. Treatment is by haemostasis and leaving the wound B. Immobilization of the wounded part helps wound
open to drain. healing.
C. Corticosteroid intake impairs wound healing.
160- Regarding the causative traumatic agent, the D. The intake of cancer chemotherapy delays wound
following statements are correct, except: healing, (All Are True)

A. The extent of tissue damage by a missile depends upon 168- About wound management, one statement only of
its weight. the following is correct:
B. The extent of tissue damage by a missile is
proportionate to its velocity. A. Debridement is the removal of dead tissues and
C. Stab wounds are inflicted by knives and may injure foreign bodies.
viscera. B. Debridement is necessary for clean incised wounds to
D. Stab wounds usually fracture bones and thus add to get good drainage.
tissue damage. C. Nerve repair in lacerated crushed wounds is of first
priority.
161- For contaminated incised wounds of 18 hours D. In fresh clean-cut wounds mark the cut tendons for
duration one choice is correct: delayed treatment.

9 EDITED BY: MOHAMED ELSHEBL


169- Factors that make lacerated wounds more liable to 177- The injuries that are induced by high velocity
infection include the following, except: missiles are attributed to all the following, except:

A. Presence of foreign bodies. A. Passage of the missile in its track through the tissues.
B. Ischemic or dead tissues. B. Sonic waves. C. Cavitational effect.
C. Suture of the wound under tension. D. Neatly slicing the tissues.
D. Leaving the wound open after debridement.
178- A keloid
170- Delayed primary closure of the skin means:
A. Is more frequently seen in black people than
A. Closure of the skin by sub-cuticular sutures. Caucasians.
B. Skin closure on the 12th day after surgery. B. Never follows a puncture wound .
C. Skin closure when pus stops to drain from the wound. C. Stops growing after 6 months.
D. Skin closure on the 5th day. D. Never develops in a sternotomy incision.

171- Postoperative wound infection can produce the 179- Positive risk factors for wound infection include:
following, except:
a- Obesity. b- Foreign body.
A. Stitch sinus. B. Reactionary hemorrhage. c- Poor surgical technique. d-All of the above.
C. Septicemia. D. Incisional hernia.
180- Regarding wound infection rates which the
172- Incised wounds: following is true:

A. Heal faster than lacerated wounds. a- The rate infection of clean wounds(e.g.hernia repair) is
B. Are more likely to injure deeper structures than missile 1-2%
wounds. b- The rate infection of Clean-contaminated (e.g.
C. The accompanying shock waves produce extensive local cholecystectomy) is<1 0%.
damage. c- The rate infection of contaminated wounds (e.g.
D. Treatment is by haemostasis and leaving the wound colectomy for obstruction) is 15-20%.
open to drain. d- All are true.

173- Regarding the causative traumatic agent, the 181- The most important cause of post-operative wound
following statements are correct, except: infection is:

A. The extent of tissue damage by a missile depends upon a- Inadequate preoperative preparation of the skin.
its weight. b- Poor local blood supply.
B. The extent of tissue damage by a missile is c- Low general resistance.
proportionate to its velocity. d- Presence of dead space.
C. Stab wounds are inflicted by knives and may injure
viscera. 182- A boil is:
D. Stab wounds usually fracture bones and thus add to
tissue damage. a- Any abscess of the skin.
b- The same as a carbuncle.
174- For contaminated incised wounds of 18 hours c- An acute infection of a hair follicle.
duration one choice is correct: d- An infection of subcutaneous tissue.

A. Wound irrigation and delayed primary closure is 183- Fournier›s gangrene of the scrotum is caused by all
preferred. EXCEPT:
B. Urgent repair of cut tendons provides the best results.
C. Immediate repair of cut nerves by nerve graft is advised: a- Clostridia. b- Bacteroids.
D. Extensive debridement of muscles is necessary. c- Coliforms d- Peptostreptococci.

175- About healing by secondary intention, all the 184- Debridement of a wound means:
following statements are true, except:
a- Excising 1 mm skin from the edges of a wound.
A. Wound infection leads to healing by secondary b- Not excising skin but excising all damaged muscle.
intention. c- Laying open all layers of a wound and excision of
B. It is a slow process. devitalised tissue.
C. It ultimately produces a neat scar. d- Delayed primary suture.
D. Wound contraction is involved in this type of healing.
185- The most common organism encountered in intra-
176- About scars, all the following are true, except: abdominal infection is:

A. Contracture impairs joint movement. a- Staphylococcus. b- Pseudomonas.


B. Squamous cell carcinoma may arise on top of a scar. c- E.coli. d-Proteus.
C. A scar regains the original tissue tensile strength in 6
months. 186- The main organism of endotoxin release in multiple
D. A superficial burn leaves no scar. organ dysfuuctiou syndrome is:

a. Proteus vulgaris. b. E. coli.


c. Pseudomonas. d. Acenetobacter.

SURGERY Q-BANK VOL. 1 10


187- The most common anaerobe in colon is: 197- Indicate the wrong statement about gas gangrene:

a. Group D streptococci. b. Cl.welchii. a- Is caused by anaerobic spore-bearing clostridia.


c. B.fragilis. d. Cl. septicum. b- Has a very short incubation period (1 - 2 days).
c- Is associated with high fever.
188- The most important factor influencing prognosis in d- May be associated with anaemia and jaundice.
multiple organ failure is:
198- The most effective method to avoid gas gangrene
a. Patient›s age. infection in a contaminated wound after a road car
b. Number of organs that have failed. accident is:
c. The initiating process.
d. Sequence in which organs failed. a. Anti-gas gangrene serum injection
b. Penicillin G injection
189- Which statement is wrong concerning clostridium c. Wound irrigation and debridement.
tetani: d. Rapid suturing of the wound

a- Is a Gram-negative bacillus. 199- Most common infection of the hand is:


b- Has a drum-stick appearance.
c- Is a strict anaerobe. a- Acute paronychia. b- Web space infection.
d- Produces highly resistant spores. c- Palmar abscess. d- Felon.

190- The tetanospasmin acts by: 200- Felon is:

a. Direct stimulation of motor endplate. a- Mid palmar space infection.


b. Stimulation of anterior horn cells. b- Terminal pulp space infection.
c. Inhibition of cholinesterase. d. B and C. c- Infection of subhyoid bursa.
d- Ulnar bursa infection.
191- The prodromal symptoms of tetanus include the
following EXCEPT: 201- Infection of the tendon sheath of the fifth finger
tends to spread ready to the:
a- Restlessness and sleeplessness. .
b- Rigidity or muscular twitches in the region of the a- Fourth digital sheath. b- Fourth web space.
wound. c- Midpalmar space. d- Ulnar bursa.
c- Stiffness and twitches of the jaw muscles.
d- High fever. 202- In the hand, infection of the mid palmar space is
most often due to:
*** 192- The poorest prognosis in tetanus is when:
a- Deep punctured wounds ofthe palm.
a. Spasms are violent and continuous. b- Direct spread from intrathecal whitlows.
b. Spasms appearing within 48 hours of trismus. c- Local spread from ulnar bursa or thenar space,
c. Contamination of large wound. d- Lymphatic spread from superficial infections.
d. Spasms brought about by skin stimulus.
203- Ballooning of the thenar eminence with abduction
193- In a patient of tetanus with severe spasms and of the thumb and marked oedema of the dorsum of the
cyanosis, the first thing to be done is: hand is diagnostic of:

a. Thorough wound debridement and heavy dose of a- Subcutaneous whitlow of the thenar eminence.
penicillin. b- Intrathecal whitlow of the thumb.
b. Human tetanus immunoglobulin. c- Radial bursitis.
c. Tracheostomy, muscle relaxant and ventilator. d- Thenar space infection.
d. Sedation and nasogastric suction.
204- Kanavel›s sign is:
194- Following active immunization with tetanus toxoid
injection, how often should a tetanus toxoid booster is a. Swelling above the flexor retinaculum.
given: b. Flexion of the thumb when the radial bursa is infected.
c. Flexion of the fingers in a compound palmar ganglion.
a- Every year. b- Every 2 years. d. Tenderness over an infected ulnar bursa between the
c- Every 5 years. d- Every 10 years transverse palmar creases.

195- Gas gangrene can be caused by which of the 205- The space of Parona is:
following:
a. In the wrist between the deep flexor tendons and the
a- Clostridium welchii (perfringcns) pronator 39uadrates.
b- Clostridium histolyticum. b. Above the patella between the quadriceps muscle and
c- Clostridium novyi. d- All of the above. the femur.
c. Beneath the tendon of the iliopsoas.
196- The most lethal exotoxin of CI. Welchii is: d. Between the Achilles tendon and the posterior aspect
of the tibia.
a- Hvaluronidase. b- Collagenase.
c- Haemolysin. d- Lecithinase.

11 EDITED BY: MOHAMED ELSHEBL


206- The ‹frog hand› is seen in: A. Urgent arteriography
B. Antibiotics
a. Acute fulminating tenosynovitis. C. Control of diabetes mellitus
b. Infection of proximal volar space. D. Wide incisions to drain pus
c. Deep palmar abscess.
d. None of the above. 216- Find the incorrect statement about prophylactic
antibiotics against postoperative wound infection:
207- False regarding carbuncle is:
A. They are indicated for contaminated operations.
a- Infective gangrene of subcutaneous tissue. B. They are indicated when a prosthetic implant is
b- Caused by staphylococcus. inserted.
c- Diabetics are more prone. C. They are started one hour before the operation or with
d- Caused by Streptococcus. induction of anaesthesia.
D. Antibiotic administration should be continued for five
208- False regarding erysipelas is: days after the operation.

a- Streptococcalinfection. 217- What is the most effective timing of a specific


b- Usually extends to deeper soft tissues. antibiotic during preparation of the patient for elective
c- Margins are raised. colectomy?
d- Positive Milan`s sign.
A. A single preoperative dose and 24 hrs postoperative
209- Factors that expose patients to increased risk for dose.
surgical site infections include: B. A single preoperative dose and 48 hrs postoperative
dose.
a- Diabetes. b- malnutrition. C. A single dose given within 30 min prior to skin
c- prolonged preoperative stay d- All of the above. incision.
D. A single dose at the time of skin incision.
210- Regarding surgical antimicrobial prophylaxis all the
following are true EXCEPT 218- The main line of treatment of an acute pyogenic
abscess of the thigh is:
a- It is necessary in clean wounds.
b- It should be given 30 minutes before skin incision. A. Repeated aspiration with a needle and local antibiotic
c- Discontinuation of the antibiotic within 24 hours after injections.
surgery is recommended. B. Parenteral antibiotics against Gram-negative bacilli.
d- Indiscriminate use of antimicrobials can lead to the C. Repeated I.V. injections of high doses of metronidazole.
development of antibiotic resistant microorganisms. D. Incision and drainage of the abscess.

211- The following measures are important contributing 219- About postoperative wound infection, one
factor s in controlling bacterial wound contamination in statement only is true:
the operating room, EXCEPT:
A. Is more frequent in thyroidectomy than with colectomy.
a- Appropriately timed preoperative antibiotic prophylaxis B. Antibiotics have no effect in its prevention.
b- Scrubbing, gowning and gloving. C. Usually presents in the first postoperative day.
c- Surgical site, hair shaving in the evening prior D. Is usually followed by secondary intention healing.
operation
d- Antimicrobial skin preparation 220- Which one of the below is not a risk for surgical site
infection (SSI)?
212- The organism that produces green pus is:
A. Recent surgery. B. Infancy.
A. Bacteroides fragilis. C. Prolonged hospitalization. D. Surgery after radiation.
B. Beta haemolytic streptococci.
C. Pseudomonas aeruoginosa. D. Salmonella. 221- About cellulitis, one statement only is correct:

213- One of the following is an anaerobic organism: A. Treatment is essentially incision and drainage of pus.
B. Infection is usually caused by Streptococci.
A. E. coli. B. Clostridium perfringens. C. Histologically the presence of giant cells is diagnostic.
C. Staphylococcus aureus. D. Pseudomonas aeroginosa. D. The antibiotic of choice is metronidazole.

214- To prevent foot infection in a diabetic patient the 222- Which one of the following is the predominant
following recommendations are all correct except cation in intracellular fluid?

A. Careful trimming of toe nails. A. Chloride. B. Calcium.


B. Monthly injection of the long-acting benzathene C. Sodium D. Potassium
penicillin.
C. Regular washing and daily inspection of the feet by the 223- Hypomagnesaemia resembles the clinical picture of
patient. which one of the following?
D. Early treatment of taenia pedis infection.
A. Hypokalaemia. B. Hyperphosphataemia.
215- The usual treatment of diabetic foot infection C. Hyperglycaemia. D. Hypocalcaemia.
includes the following except:

SURGERY Q-BANK VOL. 1 12


224- Regarding hypokalaemia, all the following A. The main source of energy is 25% dextrose.
statements are true, except: B. Amino acids can be mixed with dextrose in a 3.L bag.
C. The rate of parenteral nutrition administration is better
A. Hypokalaemia causes acidosis. regulated by a pump.
B. Severe diarrhoea may cause Hypokalaemia. D. Metabolic control is better maintained by TPN than by
C. Hypokalaemia causes muscle weakness. enteral nutrition.
D. Hypokalaemia causes paralytic ileus.
233- About intravenous lipid preparations, all the
225- About metabolic acidosis, one statement only is following statements are true, except:
true:
A. They provide energy at a ratio of 9 kcal/ 19 fat.
A. It is caused by pyloric stenosis. B. They provide essential fatty acids.
B. It causes hyperkalaemia. C. They make the main source of energy of total
C. Arterial bicarbonate level is elevated. parenteral nutrition (TPN).
D. PH is above 7.4. D. They can be given through a peripheral vein.

226- About basal daily requirements, all the following 234- HLA antigens are present on the membrane of:
are true, except:
A. B lymphocytes. B. Monocytes.
A. 0kcal/kg/day. B. 0.7g protein/kg/day. C. Macrophages. D. All nucleated cells.
C. 19 nitrogen/kg/day. D. 1mmol sodium/kg/day.
235- Rejection on the third post-operative day is
227- In a surgical patient, malnutrition causes all the considered:
following complications, except:
A. Hyperacute rejection. B. Acute rejection.
A. Incisional hernia. C. Chronic rejection. D. Accelerated acute rejection.
B. Disruption of intestinal anastomosis.
C. Raised incidence of infections. 236- What is the cold ischaemic time of the harvested
D. Thrombocytopenia. kidney?

228- All the following conditions cause hypercatabolism, A. 6 hours. B. 12 hours.


except: C. 24 hours. D. 36 hours.

A. Iliofemoral deep vein thrombosis. 237- What is the most common cause of liver failure
B. Major trauma and burns. following liver transplantation?
C. Major surgical procedures.
D. Severe acute pancreatitis. A. Chronic hepatitis. B. Metabolic disease.
C. Alcoholic cirrhosis. D. Bilharzial cirrhosis.
229- Ten days after right hemicolectomy, a patient
develops an external fistula that discharges about 2398- What is the common vascular complication
100ml/day. There is no evidence of sepsis and the patient following liver transplantation?
passes regular daily motions. The suitable method of
supplying his nutritional needs is: A. Portal vein thrombosis.
B. Hepatic artery thrombosis.
A. Oral intake of a regular diet. C. Hepatic vein thrombosis.
B. Nasogastric fluid feeding. D. Inferior vena cava thrombosis
C. NPO and jejunostomy feeding.
D. NPO and total parenteral nutrition. 239- What is the cause of post-transplant
Iymphoproliferative disease?
230- About jejunostomy feeding, all the following
statements are true, except: A. Epstein-Barr virus.
B. Overdose of immunosuppression.
A. It is a useful temporary feeding method after complex C. Poor immunosuppression.
gastric operations. D. None of the above.
B. Jejunostomy feeding starts with elemental formulas
then proceeds gradually to blenderised food. 240- About transplantation terminology:
C. Jejunostomy feeds should be sterile.
D. Jejunostomy feeding can cause colicky abdominal pain. A. Autograft is the term that is given when the same
individual is the donor and recipient.
231- The complications and disadvantages of nasogastric B. Isograft is the term that is given when tissue transfer is
tube feeding include all the following, except: from an identical twin.
C. Allograft is the term that is given when the donor
A. High cost. B. Possibility of aspiration pneumonia. and recipient are genetically dissimilar but of the same
C. Pharyngitis. D. Oesophagitis. species.
D. Xenograft is the term that is given when the donor and
232- About total parenteral nutrition (TPNL all the recipient-belong to different species (animal to man).
following statements are true, except:
(All Are True)

13 EDITED BY: MOHAMED ELSHEBL


241- About graft rejection, one statement only is true: 2- What is the most common indication for intubation in
a trauma patient?
A. Treating accelerated acute rejection with increasing
immunosuppressive dose is usually successful. (A) Altered mental status (B) Inhalation injury
B. Acute rejection is mediated by humoral immune (C) Facial injury (D) Cervical hematoma
mechanism.
C. Acute rejection can be treated with raising the dose of 3- Which of the following trauma patients with airway
immunosuppressive therapy. compromise and failed endotracheal intubation should
D. Acute rejection occurs within the first week of undergoe emergency tracheostomy (rather than
transplantation. cricothyroidotomy)?

242- The immunosuppressive agent that is known to (A) An 84 -year- old male with blunt trauma to the neck
cause renal impairment is: (B) A 65 -year- old female with a stab wound to the sub-
mandibular region
A. Cyclosporine A. B. Azathioprine. (C) A 16-year-old male with a gunshot wound to the neck
C. Prednisolone. D. Monoclonal antibody OKT3. (D) A 6-year-old female with a crush injury to the face

243- Immunosuppressive therapy may increase the 4- Which of the following is a life-threatening
incidence of which one of the following malignancies: compromise to circulation and must be identified
during the primary survey?
A. Lymphoma. B. Breast cancer.
C. Bronchial carcinoma. D. Colon cancer. (A) Unstable pelvic fracture (B) Pericardial effusion
(C) 40% pneumothorax (D) Femoral artery injury
244- Throbbing pain indicates:
5- A 27-year-old man presents to the ED after receiving
a- Abcess formation. b- Fluid under tension. blows to the head. He opens his eyeswith painful stimuli,
c- Need for drainage. d- Any of the above. is confused and localizes to pain. What is his Glasgow
Coma Score?
245- Complications of burns include the following:
(A) 13 (B) 11
a- Acute renal failure. b- Erosive gastritis. (C) 9 (D) 7
c- Paralytic ileus. d- All of the above.
6- The appropriate treatment of an asymptomatic
246- A pulsatile lump on the side of the neck in an 80 patient with a stab wound to Zone III of the neck is:
years old man is:
(A) Observation (B) CT of the neck
a- Mobile cervical lymph node. b- Branchial cyst. (C) Angiography (D) Operative exploration
c- Carotid body tumor d- Cystic hygroma
7- A 20-year-old unrestrained driver was involved in
247- Ameloblastoma is: a motor-vehicle crash. A computed tomography (CT)
of the abdomen revealed a large hematoma in the
a- Locally malignant tumor of mandible. second portion of duodenum. The rest of the abdomen
b- Malignant tumor of gum. is normal. The initial management of this duodenal
c- A type of giant cell tumor. hematoma should be:
d- A swelling in the dorsum of the tongue.
(A) Operative evacuation
248- Patient 15 years with discrete firm cervical and left (B) Nasogastric decompression, intravenous fluids, and
axillary swellings of few months duration with mild fever gradual resumption of oral diet
and itching may be: (C) Endoscopic retrograde cholangiopancreatogram
(ERCP)
a- Non-Hodgkin`s lymphoma stage I (D) Laparotomy, pyloric exclusion, and gastrojejunostomy
b- Secondries from cancer thyroid.
c- Hodgkin`s lymphoma stage II 8- A 25-year-old man fell down from his bicycle and hit a
d- TB lymphadenitis. concrete wall on his left side. An ultrasound examination
showed free fluid in the abdomen. A CT scan confirmed
a grade III splenic injury. The most important
contraindication for a nonoperative management of the
TRAUMA AND SHOCK splenic injury is:

(A) Hemodynamic instability


1- Which of the following would mandate elective
(B) Active bleeding on CT scan
intubation in a patient with a normal voice, normal
(C) Adult patient
oxygen saturation and no respiratory distress?
(D) Lack of availability of blood for transfusion
(A) Airway bleeding
(B) Stab wound to the neck with mild swelling in the left
lateral neck
(C) Localized right lateral subcutaneous emphysema
(D) Bilateral mandibular fracture

SURGERY Q-BANK VOL. 1 14


9- An 18-year-old man is brought to the emergency (A) Clamp the bleeding artery with a vascular clamp.
department with a stab wound just to the right (B) Apply a tourniquet 7.5 cm above the wound.
of the sternum in the sixth intercostal space. His (C) Apply direct pressure with sterile gauze.
blood pressure is 80 mm Hg. Faint heart sounds and (D) Insert central venous access line.
pulsus paradoxus are noted. Auscultation of the right
chest reveals decreased breath sounds. The initial 15- The injury most often missed by selective
management of this patient should be which of the nonoperative management of abdominal stab wounds is
following? to which of the following?

(A) Aspiration of the right chest cavity (A) Colon (B) Spleen
(B) Aspiration of the pericardium (C) Ureter (D) Diaphragm
(C) Echocardiogram
(D) Insertion of central venous access line 16- A 56-year-old male is burned while sleeping in
his home. His right upper and lower extremity and
10- A 25-year-old man is shot in the left lateral chest. In the anterior aspect of the upper chest have extensive
the emergency department, his blood pressure is 120/90 second-degree burns. A second-degree burn is
mm Hg, pulse rate is 104 beats per minute (bpm), and characterized by which of the following?
respiration rate is36 breaths per minute. Chest x-ray
shows air and fluid in the left pleural cavity. Nasogastric (A) Coagulative necrosis extending to subcutaneous fat
aspiration reveals blood-stained fluid. What is the best (B) Pearly white appearance
step to rule out esophageal injury? (C) Anaesthetic
(D) Erythema and bullae formation
(A) Insertion of chest tube
(B) Insertion of nasogastric tube 17- A 17-year-old girl presents to the emergency
(C) Esophagogram with gastrografin department with a stab wound to the abdomen and a
(D) Esophagoscopy blow to the head that left her groggy. Her blood pressure
is 80/0 mm Hg, pulse is120 bpm, and respiration rate is
11- A 32-year-old female falls from the tenth floor of her 28 breaths per minute. Her abdomen has a stab wound
apartment building in an apparent suicide attempt. in the anterior axillary line at the right costal margin.
Upon presentation, the patient has obvious head Two large-bore intravenous lines, a nasogastric tube, and
and extremity injuries. Primary survey reveals that a Foley catheter are inserted. The blood pressure rises to
the patient is totally apneic. By which method is the 85 mm Hg after 2 L of Ringer’s lactate. The appropriate
immediate need for a definitive airway in this patient management is which of the following?
best provided?
(A) Peritoneal lavage (B) Ultrasound of the abdomen
(A) Orotracheal intubation (C) Exploratory laparotomy (D) CT of the head
(B) Nasotracheal intubation
(C) Percutaneous cricothyroidotomy 18- Trauma deaths most commonly occur at three
(D) Intubation over a bronchoscope distinct time periods after injury. Which of the following
statement(s) is/are true concerning the time pattern of
12- A 20-year-old woman presents to the emergency trauma mortality?
department with a stab wound to the abdomen.
There is minimal abdominal tenderness. Local wound (A) Only 10% of trauma deaths occur within seconds or
exploration indicates that the knife penetrated minutes of the injury
the peritoneum. What is the ideal use of antibiotic (B) A second mortality peak occurs within hours of injury
administration? with deaths in this time period being markedly reduced
with the development of trauma and rapid transport
(A) Preoperatively systems
(B) Intraoperatively, if a colon injury is fundo (C) Death one day to weeks after the injury are almost
(C) Postoperatively, if the patient develops fever entirely due to infection and multiple organ failure
(D) Postoperatively, based on culture and sensitivity of (D) Late mortality in trauma patients, occurring days to
fecal contamination found at the time of surgery weeks after the injury, has not been affected by better
trauma delivery systems
13- A 26-year-old man is stabbed in the right intercostal
space in the midclavicular line and presents to the 19- A 22-year-old woman presents to the emergency
emergency department. On examination, subcutaneous department with a chief complaint of severe left
emphysema of the right chest wall, absent breath upper quadrant (LUQ) pain after being punched by her
sounds, and a trachea shifted to the left are noted. What husband. Her blood pressure is 110/70 mm Hg, pulse is
is the most likely serious diagnosis? 100 bpm, and respiration rate is 24 breaths per minute.
The best means to establish a diagnosis is which of the
(A) Pneumothorax following?
(B) Tension pneumothorax
(C) Massive hemothorax (A) FAST (B) Physical examination
(D) Hemopneumothorax (C) CT of the abdomen (D) Peritoneal lavage

14- A 34-year-old man is brought into the emergency


department with a large open knife wound to the left
thigh. The patient’s systolic blood pressure is90 mm Hg.
Blood is spurting from the wound. What is the initial
management step?

15 EDITED BY: MOHAMED ELSHEBL


20- A70-year-old woman is hit by a car and injures her 27- About triage for mass casualties, all the following
midabdomen. The best way to rule out a rupture of the statements are true, except:
second part of the duodenum is by which mode?
A. Is used to sort trauma victims› priority for management
(A) Repeated physical examinations at the scene of accident.
(B) Ultrasound B. Is used to sort trauma victims› priority for transfer to
(C) Repeated amylase levels hospital.
(D) CT with oral and intravenous contrast C. Is used to sort trauma victims› priority for management
in emergency room.
21- An 18-year-old man is brought to the emergency D. Those who are likely to live, regardless of whatever
department after falling down a flight of stairs and care they receive, should be given first priority.
losing consciousness for 3 minutes. A cervical collar is
in place. The cervical spine is considered to be free of 28- For a road-traffic-accident victim, the first priority of
serious injury following which procedure? management is given to which of the following injuries?

(A) A physical examination revealing no pain or tenderness A. Open pneumothorax on the right side.
(B) A lateral cervical spine x-ray B. Tension pneumothorax on the left side.
(C) Anteroposterior (AP), lateral, and odontoid views of C. Bleeding scalp laceration.
the neck D. Fracture of the shaft of right femur.
(D) Flexion and extension views of the neck
29- A trauma victim is hypotensive. Any of the following
22- A 70-year-old man is brought into the emergency can be the cause, except:
department following his injury as a passenger in a car
crash. He complains of right side chest pain. Physical A. Extradural haematoma. B. Lacerations of the spleen.
examination reveals a respiratory rate of 42 breaths per C. Pelvic fractures. D. Liver lacerations.
minute and multiple broken ribs of a segment of the
chest wall that moves paradoxically with respiration. 30- About diagnostic peritoneal lavage (DPL) for blunt
What should the next step be? abdominal trauma, all the following statements are true,
except:
(A) Tube thoracostomy (B) Tracheostomy
(C) Thoracentesis (D) Intercostal nerve blocks A. It is a good method for diagnosing intestinal injuries.
B. It is a good method for diagnosing renal injuries.
23- A30-year-old man is brought to the emergency C. It is a good method for diagnosing intra-peritoneal
department in respiratory distress following a shotgun bleeding.
wound to the face. There is a possible cervical spine D. There is no need for it in the shocked patients who has
injury. Which is the best way to gain rapid control of the abdominal tenderness and rigidity.
airway?
31- A 32 years old man was stabbed in the central
(A) Nasotracheal intubation abdomen. He presents to the emergency room with the
(B) Cricothyroidotomy knife blade still embedded in the abdomen. His pulse
(C) Endotracheal intubation is 125/min, his blood pressure is 80/40mmHg. All the
(D) Aspiration of blood from pharynx and jaw thrust following measures are correct, except:

24- A 40-year-old woman is brought to the emergency A. Remove the knife immediately in the emergency
department following a car crash in which she was the room.
driver. In the emergency department, her blood pressure B. Place IV cannulas and start administration of lactated
is 80/60 mm Hg, pulse is 128 bpm, and respiratory Ringer's solution.
rate is 32 breaths per minute. She complains of right C. Cross-match blood.
lower chest wall and severe right upper quadrant D. No need for imaging studies of the abdomen.
(RUQ) tenderness. Her breath sounds are questionably
diminished. The immediate priority is to perform which 32- A victim of a road traffic accident is brought to the
of the following? emergency room with increasing dyspnoea and chest
pain. He is cyanotic, his neck veins are congested, his
(A) Peritoneal lavage blood pressureis90/60mmHg, and his right chest is
(B) Chest x-ray bulging and exhibits marked resonance. The most
(C) CT scan of chest and abdomen probable diagnosis is:
(D) Thoracentesis with an 18-gauge needle
A. Massive haemothorax. B. Flail chest.
25- The following statement about keloid is true ? C. Open pneumothorax. D. Tension pneumothorax.

(A) they do not extend into normal skin 33- About tension pneumothorax, one of the following
(B) local recurrence is common after excision statements is true:
(C) they often undergo malignant change
(D) they are more common in whites than in blacks A. Diagnosis should be established with Plain chest X-ray.
B. Diagnosis should be confirmed with CT scan of the
26- The least likely trauma to cause multiple injuries is: chest.
C. The condition is less serious than open pneumothorax.
A. Fall from a height. B. Machinery accidents. D. Immediate treatment is placement of wide-bore
C. Road traffic accidents D. Explosions. needles in the ipsilateral hemithorax.

SURGERY Q-BANK VOL. 1 16


34- By the end of an exploratory laparotomy, the surgeon 42- About shock, one statement only is true:
finds that the viscera are bulging and it is difficult to
close the abdomen. The best to be done is: A. In hypovolaemic shock, perfusion of different organs is
simultaneously affected.
A. Using a polygalactin mesh for closure. B. There is metabolic alkalosis.
B. Using a polypropylene mesh for closure. C. Cardiac output is reduced in cardiogenic shock.
C. Leaving the abdomen open and covering the viscera D. Severe arrhythmias cause hypovolaemic shock.
with sterile packs.
D. Closing the abdomen under tension. 43- About consequences of shock, all the following
statements are true, except:
35- About Glasgow Coma Scale (GCS) all the following
statements are true, except: A. Hypoxia reduces energy production.
B. Sodium/potassium pump is impaired.
A. It is a numerical measure of the level of consciousness. C. Excess sodium leaks out of cells.
B. A rise in intra-cranial tension reduces GCS. D. Endothelial cells leak protein-rich fluid from the
C. GCS components are eye opening, best verbal response capillaries.
and best motor response.
D. The highest GCS is 12. 44- About the compensatory mechanisms of
hypovolaemia, all the following are true, except:
36- A trauma victim opens his eyes in response to
commands, is disoriented when asked about his A. The earliest mechanism is the secretion of aldosterone.
profession, and localizes painful stimuli. His Glasgow B. The sympatho-adrenal response is mediated by the
Coma Scale is: baroreceptors.
C. The sympatho-adrenal response results in tachycardia.
A. 6. B. 8. C. 9. D. 12. D. The sympatho-adrenal response increases the stroke
volume.
37- In a bleeding patient, the most important parameter
to assess successful fluid replacement is: 45- About management of shock, all the following
statements are true, except:
A. Pulse rate. B. Blood pressure.
C. Urine output. D. Skin elasticity. A. Oxygen administration is indicated for all shocked
patients.
38- About compartment syndrome of the leg, all the B. Immediate volume replacement in haemorrhagic
following statements are true, except: shock is best done by lactated Ringer›s solution.
C. Central venous pressure reflects preload.
A. It may result from contusions and hematomas of leg D. Dopamine reduces renal blood flow.
muscles.
B. It may result from compound fractures of the tibia. 46- Shock with elevated CVP is:
C. The leg feels tense.
D. It may cause arterial insufficiency. A. Hypovolaemic shock. B. Septic shock.
C. Cardiogenic shock. D. Neurogenic shock.
39- A workman fell on a bar that hit the perineum. On
presentation in the emergency room he is found to have 47- Shock with bradycardia is:
a drop of blood at the tip of the urethra. There is also
a perineal haematoma. Management includes all the A. Hypovolaemic shock. B. Septic shock.
following, except: C. Cardiogenic shock. D. Neurogenic shock.

A. Advise the patient not to pass urine. 48- In septic shock, one statement only is true:
B. Immediate catheterization.
C. Antibiotic administration. A. The overall mortality rate is about 5%.
D. Urgent ascending urethrography. B. Gram-negative organisms are involved exclusively.
C. Septic shock is a maldistribution shock.
40- About early management of fall-from-height victims, D. In its hypodynamic phase the skin is flushed.
all the following statements are true, except:
49- A shocked 70-year-old man had urgent laparotomy
A. A cervical collar should be postponed until cervical for neglected duodenal ulcer perforation. At the end of
spine fracture is proved. the operation, excessive undue bleeding occurs from
B. Arrest of external bleeding by compression takes the wound edges and from the cannula site. The most
priority over splinting of fractures. probable cause is:
C. Cervical spine X-ray is a routine part of early
management. A. Consumption of coagulation factors
D. Chest X-ray is a routine part of early management. B. Dilution of coagulation factors;
C. Hvpoprothrombinaemia that is caused by liver failure.
41- Regarding traumatic diaphragmatic rupture, all the D. Haemophilia.
following statements are true, except:
50- All the following are features of ARDS, except:
A. It mainly affects the left side.
B. Plain Chest X-ray can be diagnostic. A. Increased lung compliance.
C. It is very easy to diagnose. B. Surfactant deficiency.
D. It is likely to be associated with abdominal and thoracic C. Hypoxia.
injuries. D. Pulmonary infiltrates on chest X-ray.

17 EDITED BY: MOHAMED ELSHEBL


51- The following statements about ARDS are all true, 3- Galactorrhea, a milky discharge from the nipple in
except: nonpregnant women, is most likely to be associated
with which of the following?
A. Major sepsis, burns and severe pancreatitis are
important causes. (A) Fibroadenoma (B) Tubular adenoma
B. Dyspnea is the main symptom. (C) Pituitary adenoma (D) Breast abscess
C. There is impairment of ventilation and perfusion but
not diffusion. 4- A 28-year-old female figure skater presents several
D. Clinical manifestations precede the appearance of weeks after having sustained an injury to her left breast.
radiological signs. She has a painful mass in the upper outer quadrant.
Skin retraction is noticed, and a hard mass, 3–4 cm in
52- To assess head injury in 1ry survey, we use: diameter, can easily be palpated. What is the most likely
diagnosis?
a- Glasgow coma scale.
b- Modified Glasgow coma scale. (A) Infiltrating carcinoma (B) Breast abscess
c- AVPU system. (C) Hematoma (D) Fat necrosis
d- None of the above.
5- A 35-year-old patient presents to your office with
53- To arrest bleeding from right thigh at the scene of chronic draining subcutaneous periareolar abscesses,
the accident, we do the following: which have been incised and drained many times in
the past 5 years but keep recurring. What is the best
a- Application of tourniquet. treatment of choice?
b- Direct compression.
c- Usage of hemostatic agent. (A) Repeat incision and drainage (I and D) since the
d- Try to ligate the bleeding vessel. previous procedures were inadequate
(B) Long-term antibiotics
54- Amputated finger will be preserved by: (C) Complete excision of the drainage tract
(D) Tell the patient there is nothing to do and that this will
a- Double layered system with saline in the inner one eventually resolve with age
and ice in the outer one.
b- Warm water. 6- A patient presents 1 month after a benign right breast
c- Formalin. biopsy with a lateral subcutaneous cord felt just under
d- Box filled with ice cubes. the skin and causing pain. The etiology of this condition
is?
55- The technique used to clear the victim›s airway is:
(A) Fat necrosis
a- Lift chin up, tilt head back. (B) Infection
b- Push chin down, tilt head forward. (C) Superficial thrombophlebitis
c- Lift chin up, turn head sideways. (D) Misdiagnosed breast cancer
d- None of the above.
7- A 36-year-old woman complains of a 3-month history
56- The most common airway obstruction is: of bloody discharge from the nipple. At examination,
a small nodule is found, deep to the areola. Careful
a- Dentures. b- Food. palpation of the nippleareolar complex results in
c- The tongue. d- None of the above. blood arrearing at the 3 O’clock position. Mammogram
findings are normal. What is the likeliest diagnosis?

BREAST SURGERY (A) Intraductal papilloma


(C) Carcinoma in situ
(B) Intraductal carcinoma
(D) Fat necrosis

1- A 35-year-old professional dancer presents with a 8- During a routine screening mammography, a 62-year-
well-defined, tense, smooth mass in the upper outer old teacher is informed that she has changes on her
quadrant of the left breast. She states that the mass mammography, and she should consult her physician.
becomes larger just before onset of her periods. She can be reassured that the findings that indicate a
Aspiration yields a clear yellow fluid and the mass benign condition are which of the following?
disappears. The most likely diagnosis is:
(A) Discrete, stellate mass
(A) Fibroadenoma is a cyst. (B) Fine, clustered calcifictions
(B) Fibrocystic disease of the breast. (C) Coarse calcifications
(C) Carcinoma in a cyst. (D) Solid, clearly defined mass with irregular edges
(D) Galactocele
9- A 40-year-old lawyer comes into your office after
2- An 18-year-old presents with a wellcircumscribed2-cm seeing some information on the Internet relating to
mass in her right breast. The mass is painless and has breast cancer. Which of the following factors has not
a rubbery consistency and discrete borders. It appears shown to increase a woman’s risk for breast cancer?
to move freely through the breast tissue. What is the
likeliest diagnosis? (A) Smoking
(B) Atypia seen on pathology from previous breast biopsy
(A) Carcinoma (B) Cyst (C) First-degree relative with history of breast cancer
(C) Fibroadenoma (D) Cystosarcoma phyllodes (D) Increasing age

SURGERY Q-BANK VOL. 1 18


10- A 53-year-old waitress inquires about the (A) IIIC (B) IV
implications of positive estrogen receptors (ER+) in an (C) IIB (D) IIIB
invasive carcinoma that is excised from her left breast.
She should be informed of what? 16- A65-year-old woman undergoes a lumpectomy
and sentinel lymph node biopsy and is found to have a
(A) They are more often positive in patients under 50 years 5-mm tubular cancer ER and PR positive and a negative
of age. sentinel lymph node. What adjuvant treatment should
(B) If the receptors are positive, antiestrogen therapy is not be recommended?
indicated.
(C) If the receptors are positive, the prognosis is more (A) Chemotherapy and radiation
unfavorable. (B) Radiation treatment only
(D) ER and progesterone receptor (PR) status should be (C) Hormonal therapy only
determined in all cases of breast carcinoma. (D) Radiotherapy and hormonal therapy

11- A52-year-old undergoes a left modified radical 17- A 41-year-old patient presents to your office with
mastectomy for a 2-cm breast cancer. She should be a biopsy proven invasive ductal cancer in the upper
informed that the factor which has the greatest impact outer aspect of her left breast, a suspicious palpable
on her prognosis is? left axillary lymph node, and diffuse calcifications
throughout the rest of the breast proven to be DCIS on
(A) The size of the primary tumor stereotactic biopsy. The best surgical option is:
(B) The histological type of the carcinoma
(C) The number of axillary nodes positive for metastasis (A) Modified radical mastectomy
(D) Positive findings on tests for the presence of the BRCA (B) Simple mastectomy
(breast cancer)1 gene (C) Lumpectomy with sentinel lymph node biopsy
(D) Radical mastectomy
12- A 46-year-old woman presents with a mammogram
that shows a 1-cm cluster of fine calcification in the right 18- A premenopausal 44-year-old woman undergoes
breast. Following mammographic wire localization, the a quadrantectomy and node dissection for a 2-cm
lesion is excised and the pathology reported as ductal infiltrating carcinoma of the left breast. The margins are
carcinoma in situ (DCIS) with comedo features and free clear, and 5 out of 15 lymph nodes are involved. ER and
margins. What advice should be given to the patient? PR are positive. Recommended adjuvant therapy should
include which of the following?
(A) If untreated, about 30% of such lesions become
invasive over a 10-year period. (A) Radiotherapy alone
(B) Comedo DCIS is less aggressive than noncomedo (B) Estrogen therapy alone
DCIS. (C) Modified radical mastectomy
(C) Bilateral mastectomy and radiotherapy are the (D) Chemotherapy, radiotherapy, and tamoxifen
preferred treatments.
(D) Axillary node dissection is always indicated. 19- An 18-week pregnant, 35-year-old woman presents
after undergoing a modified radical mastectomy for
13- A 43-year-old premenopausal patient has a biopsy a 2-cm ductal cancer with one out of fifteen positive
showing focal lobular carcinoma in situ (LCIS) in the area axillary lymph nodes. What should she be informed of
of calcification. With regard to the LCIS, you should tell regarding breast cancer during pregnancy?
the patient which of the following?
(A) She cannot undergo chemotherapy until after she
(A) She needs a simple mastectomy. delivers.
(B) She must be placed on tamoxifen and chemotherapy. (B) She should have a therapeutic abortion in order to
(C) This is a premalignant lesion, and she requires proceed with radiotherapy.
additional lumpectomy and radiotherapy. (C) Breast cancer is the most common cancer during
(D) She is at increased risk of breast cancer, and she pregnancy.
should just be observed closely. (D) Most of these cancers are ER+.

14- A partially blind 65-year-old mother presents with 20- After undergoing modified radical mastectomy for
a slight change in color of the areola of her left breast. cancer of the right breast, a 52-year-old female teacher
An eczematous rash of the left areola has persisted for becomes aware that the medial end of her scapula
the last 3 months. Biopsy of the nipple reveals Paget’s becomes prominent in protraction movements at the
disease. In Paget’s disease of the nipple which of the shoulder. She also complains of some weakness in
following is TRUE? complete abduction of the same shoulder. What nerve
was injured?
(A) Carcinoma of the breast is rarely found.
(B) Surgical therapy often fails to cure Paget’s disease (A) Long thoracic
(C) The diagnosis should be made by nipple biopsy when (B) Thoracodorsal
suspected. (C) Median
(D) Paget’s disease of the bone is commonly encountered. (D) Intercostobrachial

15- A 39-year-old patient presents to your office with a


left 3.5-cm breast tumor, which on core needle biopsy,
is shown to be an invasive ductal cancer. On left axillary
examination, she has a hard nonfixed lymph node.
A biopsy of a left supraclavicular node is positive for
malignancy. Her stage is currently classified as?

19 EDITED BY: MOHAMED ELSHEBL


21- A 50-year-old patient has recently undergone a D. Mammography is indicated if an abscess is suspected
mastectomy for a 2.5 -cm multicentric breast cancer
with three positive axillary lymph nodes (stage IIB). 28- The following statements about fibrocystic breast
A metastatic survey is done, and is negative, and she are all true, except:
receives adjuvant chemotherapy. The most common site
for distant metastasis would be: A. May be asymptomatic
B. May cause nipple retraction.
(A) Brain (B) Bone C. Is a cause of breast pain
(C) Lung (D) Liver D. Produces breast nodularity.

22- A 45-year-old premenopausal woman undergoes 29- Treatment of cyclic mastalgia may include all the
a left breast lumpectomy for a 1.5-cm, lymph node following, except:
positive, hormone sensitive invasive breast cancer.
She receives chemotherapy, radiotherapy, and is on A. Exclusion of malignancy and reassurance
tamoxifen. Recommended follow-up after therapy B. Avoidance of breast support
should always include: C. Giving up caffeine consumption
D. Bromocriptine 2.5 mg bid
(A) Blood tumor markers drawn every3–6 months after
treatment. 30- The following statements about fibroadenoma of the
(B) Routine monitoring of liver function tests (LFTs) every breast are all true, except:
3–6 months after treatment.
(C) Yearly bone scans. A. Usually affects females in the second and third decades
(D) Routine clinical examination every3–6 months for the of life.
first 5 years after treatment as well as continued yearly B. May be multiple.
mammography. C. Freely mobile.
D.If left untreated it turns malignant in 9% of cases
23- A 56-year-old male patient develops an accentric within 15 years.
hard breast lump over the past few months and a biopsy
proves this to be breast carcinoma. Of all breast cancers, 31- Regarding breast cysts, one statement is true:
the rate of occurrence in males is which of the following?
A. The commonest is galactocele.
(A) <1% (B) 4% B. Malignancy is suspected if the cyst is tense.
(C) 10% (D) >10% C. Malignancy is suspected if the lump does not totally
disappear after complete aspiration of fluid.
24- A 25-year-old nonalcoholic man has noticeable right D. Multiplicity of cysts raises suspicion of malignancy.
gynecomastia since age 20. He is most uncomfortable
and reluctant to swim or exercise at a gym for fear of 32- The following statements about mammary duct
being an object of derision. He should be advised to have ectasia are all true, except:
which of the following?
A. May produce a mass similar to carcinoma in the
(A) Right mastectomy axillary tail of the breast.
(B) Observation B. May produce non-lactational mastitis and abscess.
(C) Needle biopsy of the breast C. May produce a mamillary fistula.
(D) Endocrine workup and right subcutaneous D. Anaerobic bacteria are commonly implicated.
mastectomy
33- The commonest histological type of breast cancer is:
25- A 36-year-old woman presents with a substantial
unilateral breast enlargement. She had presumed A. Duct carcinoma in situ.
that this was normal, but on examination, a large, firm B. Lobular carcinoma in situ.
tumor is palpated by the attending physician. There C. Invasive duct carcinoma.
is early erosion on the skin. Afavorable outlook can be D. Invasive lobular carcinoma.
anticipated if the lesion is which of the following?
34- A woman has a 3 cm breast mass. There are neither
(A) Sarcoma palpable axillary lymph nodes nor clinical evidence of
(B) Cystosarcoma phyllodes metastasis. FNAC proves the presence of invasive duct
(C) Infiltrating carcinoma carcinoma. TNM clinical staging is:
(D) Inflammatory carcinoma
A. T1 Nl Ml B. T1 N0 M0
26- The axillary lymph nodes are divided into three levels C. T2 N2 M0 D. T2 N0 M0
I, II and III by
35- Breast self-examination:
A. The axillary artery. B. The axillary vein.
C. The pectoralis minor muscle. D. The clavicle. A. Is recommended for all women above the age of
twenty.
27- About acute lactational breast abscess, one B. Is recommended for women above the age of 40 only.
statement only is true: C. Is recommended to be done monthly in the immediate
premenstrual period.
A. Is commonly multilocular. D. Is recommended to be done twice yearly.
B. Weaning the baby is mandatory
C. Aspirating pus by a wide bore needle is superior to
incisional drainage as it avoids injury to lactiferous ducts

SURGERY Q-BANK VOL. 1 20


36- The following statements about mammography are A. May be discovered on screening mammography for
all true, except: asymptomatic women.
B. Is not a true malignancy.
A. Indicated for the diagnosis of a breast lump. C. It does not progress to invasive cancer but is a powerful
B. Indicated for regular screening of women who are at marker of increased cancer risk.
high-risk of development of breast cancer. D. Close observation is all that is needed.
C. Indicated for follow-up of contralateral breast after
mastectomy for breast cancer. 44- Early breast cancer:
D. More informative in young women below the age of
35 years. A. Means a T1 N1 M0 tumor or less.
B. Means that cure is impossible.
37- The following statements about breast cancer C. Means that microscopic metastases are not present.
prognosis are all true, except: D. Primary treatment is by surgery ± radiotherapy.

A. Is worsened by the detection of axillary node deposits. 45- For modified radical mastectomy all the following
B. Is better in estrogen receptor +ve cases. statements are true, except:
C. Is better for tumours in the medial than in the lateral
half of the breast. A. The areola and nipple are removed.
D. Is worse for invasive duct carcinoma than for B. The whole breast tissue is removed.
cystosarcoma phylloides. C. Block dissection of the axillary nodes requires excision
of axillary vein.
38- The following statements about Paget›s disease of D. Nerve to serratus anterior should be spared.
the nipple are all true, except:
46- The following statements about post-mastectomy
A. Is lobular carcinoma arising at the opening of a arm oedema are all true, except:
lactiferous duct on the nipple.
B. Should be differentiated from eczema of the nipple. A. Is always transient.
C. Sometimes there is no palpable mass. B. Adding radiotherapy to the axilla after clearance of
D. Mastectomy is indicated. lymph nodes increases the risk of oedema.
C. Rough dissection of the axillary vein at surgery is one of
39- Regarding axillary node involvement in cancer of the the causes.
breast, 511 the following statements are true, except: D. Insertion of an IV line in the ipsilateral upper limb is a
predisposing factor.
A. Clinical assessment of the axilla is commonly
inaccurate. 47- Acute hypercalcaemia in a patient with advanced
B. Prognosis is affected by the number of affected nodes. breast cancer:
C. Is an indication for adjuvant chemotherapy in early
cases? A. Is caused by parathyroid hyperplasia.
D. Is an indication for adding irradiation of the axilla after B. Is better treated at home.
its surgical evacuation? C. Bilateral lower limb oedema is a constant feature.
D. IV fluid administration is an important line of
40- The following statements about pulmonary treatment.
metastases of breast cancer are all true, Except:
48- Neo-adjuvant therapy for malignancy means:
A. May be asymptomatic.
B. May produce persistent cough. A. Treatment with modern methods.
C. May produce dyspnoea. B. Giving chemo and/or radiotherapy before surgery.
D. If localized to one lung while the other lung shows C. Giving chemotherapy after radical excision.
good function, pneumonectomy is indicated. D. Giving radiotherapy after palliative excision.

41- Haematogenous spread of breast cancer may 49- When screening females for breast carcinoma what
produce any of the following, except: is the most significant risk?

A. Brain metastases B. Fracture of the spine A. Three previous breast biopsies in premenopausal
C. Pleural effusion D. Hypocalcaemia females.
B. More than 2 first degree relatives with ovarian or
42- The following statements about lymphatic drainage breast carcinoma.
of the breast are all true, except: C. Hyperplasia in breast biopsy.
D. None of the above.
A. Lymph from medial half of the breast may drain in
internal mammary nodes. 50- Breast lymphatics drain into:
B. From anywhere in the breast lymphatics drain mostly
to axillary nodes. a- Axillary nodes. b- Internal mammary nodes.
C. Division of axillary nodes into levels I, II, and III depends c- Pectoral and subscapular nodes. d- All of the above.
upon the anatomical relationship with the pectoralis
minor muscle.
D. Level one axillary nodes is the highest in the axilla.

43- Regarding duct carcinoma in situ of the breast, one


statement only is true:

21 EDITED BY: MOHAMED ELSHEBL


51- Which of the following advantages breast ultrasound 61- The treatment of choice for cystosarcoma phyllodes
enjoys: is:

a- Can localize impalpable breast lumps. a. Radical mastectomy. b. Simple mastectomy.


b- Can distinguish solid from cystic lesion. c. Radiation therapy. d. Hormonal ablation.
c- Better yield in young dense breast than mammogram.
d- All of the above. 62- Mondor's disease is:

52- Mammographic lesions that are strongly associated a. An obscure type of thrombophlebitis particularly
with malignancy include all of the following EXCEPT: affecting veins of the breast.
b. Lymphoedema of the arm.
a- Large and coarse calcifications. c. Chondritis of a costal cartilage.
b- Thickened epidermis. d. Pectus excavatum.
c- Poorly defined mass lesions.
d- Fine stapled calcifications. 63- Non-malignant conditions of the breast include all
the following EXCEPT:
53- Which of the following about MRI breast imaging is
True: a. Cystosarcoma phyllodes, b. Duct ectasia.
c. Giant fibroadenoma. d. Paget's disease of the nipple.
a- Distinguishes scar from recurrence.
b- Gold standard for imaging with breast implants. 64- Protective factors for breast cancer include all
c- Management of axilla in breast cancer and recurrent EXCEPT:
disease.
d- All of the above. a. Nulliparity. b. Breast feeding.
c. Late menarche. d. Early menopause.
54- All are True of breast FNAC EXCEPT:
65- Factors associated with increased relative risk of
a- Safe and reliable. breast cancer include all EXCEPT:
b- Least invasive for cell diagnosis.
c- Very accurate. a. Nulliparity. b. Menopause before 40
d- Invasive cancer can be diagnosed from in situ disease. c. Fibrocystic disease with epithelial proliferation.
d. Early menarche.
55- What is True about breast abscess:
66- Common site of breast cancer is:
a- Rare during lactation.
b- Usually unilocular. a. Upper inner quadrant. b. Upper outer quadrant.
c- Best treated with antibiotics. c. Lower inner quadrant. d. Lower outer quadrant.
d- Requires incision and break up of all trabeculae.
67- The risk of bilateral breast cancer is HIGHEST if the
56- What is common consensus about breast abscess: first breast shows:

a- Fluctuation is late sign. a. Inflammatory carcinoma. b. Lobular carcinoma.


b- Area of induration is sector shaped. c. Medullary carcinoma. d. Infiltrating ductal carcinoma.
c- Must be drained if no resolution within 48 hours of
antibiotics. 68- Regarding Ductal carcinoma is situ (DClS) of the
d- All of the above breast all the following arc true EXCEPT:

57- Breast mouse refers to: a- Can be distinguished from invasive carcinoma on fine-
needle aspiration Cytology (FNAC).
a. Fibroadenosis. b. Fibrocystic disease. b- Accounts for around 15-20% of screen-detected breast
c. Hard fibroadenoma. d. All of the above. cancers.
c- Is associated with axillary node metastases in 1 % of
58- Most common cause of blood discharge from nipple cases.
is: d- Can be treated by mastectomy or lumpectomy with or
without radiotherapy
a- Fibroadenoma. b- Fibro-adenosis.
c- Ductal papilloma. d- Ductal carcinoma. 69- Regarding Lobular carcinoma in situ (LCIS) all are
true EXCEPT:
59- Greenish discharge from nipple indicates:
a- Multi local, multicentric and affects both breasts
a- Carcinoma. b- Duct papilloma. b- Is a marker of an increased risk for developing invasive
c- Duct carcinoma. d- Fibroadenosis. breast cancer
c- Has a characteristic mammographic appearance.
60- Massive swellings of the breast include the following d- LCIS found incidentally in breast biopsy taken for
EXCEPT: another indication.

a. Cystosarcoma phyllodes 70- The size of a stage IIIa breast tumor is:
b. Atrophic scirrhous carcinoma.
c. Diffuse hypertrophy. a. 2cm. b. 2-5 cm.
d. Giant fibroadenoma. c. > 5 cm. d. All of the above.

SURGERY Q-BANK VOL. 1 22


71- In the management of early breast cancer all the 78- Lymphatic obstruction of advanced breast cancer
following are true EXCEPT: leads to:

a- Mastectomy is associated with significantly higher a. Peau d'orange, b. Oedema of arm.


survival rates than conservative surgery. c. Lymphangiosarcoma. d. All of the above.
b- Postoperative radiotherapy reduces the incidence of
loca l recurrence. 79- What is not true of inflammatory carcinoma:
c- Adjuvant tamoxifen improves survival in both pre - and
postmenopausal patients for ER a. Highly aggressive. b. Very common.
+ve tumours. c. Breast is painful, warm and edematous.
d- Adjuvant Systemic chemotherapy has reduced d. Mistaken for mastitis.
mortality by 30%.
80- In carcinoma of breast, a secondaries deposit is most
72- Absolute Contraindications of Breast-Conserving common in:
Therapy includes all the following EXCEPT:
a. Bone b. Brain. c. Liver d. Lung.
a- Pregnancy in the first and second trimester.
b- Multicentric disease with two or more gross tumors in 81- In carcinoma of the breast, most common site of
separate quadrants. bony metastasis is:
c- Diffuse microcalciflcations that are malignant-
appearing. a- Sacral vertebra. b- Lumbar vertebra.
d- DCIS discovered by mammography. c- Thoracic vertebra d- Cervical vertebra.

73- The acronym QUART stands for: 82- Which of the following genes predisposes to breast
cancer:
a. Quadrantectomy and radiotherapy.
b. Quadrantectomy, axillary dissection and radiotherapy. a- BRCA1. b- BRCA2
c. Quadrant resection and chemotherapy. c- TP53 d- All of the above
d. None of the above.
83- Causes of gynaecomastia include:
74- What is not true about role of axillary surgery in
carcinoma of breast: a- Idiopathic b- Liver failure
c- Leprosy d- All of the above
a. Axillary lymph node metastasis is best marker for
prognosis. 84- Regarding male breast cancer spot the wrong
b. Treatment of axillary lymph nodes positively statement:
influences survival.
c. Axillary surgery should not be combined with a- It is about 1 % of all cases of breast cancer
radiotherapy to axilla. b- Predisposing causes include gynaecornastia
d. All are true. c- Caused by excess endogenous and exogenous
oestrogen
75- Five Benefits of Tamoxifen in breast carcinoma d- It has a better prognosis than the female breast
include all the following EXCEPT: cancer.

a. Tamoxifen can reduce recurrence. 85- Breast cancer may cause the following skin lesions
b. Tamoxifen can halt the progression of metastatic breast except:
cancer.
c. Tamoxifen can reduce the risk of cancer in the other a- Skin nodules. b- Peau d`orange
breast. c- Fungation d- Kaposi sarcoma.
d. Decrease incidence of endometrial carcinoma.
86- Involvement of ipsilateral axillary lymph node in
76- Regarding radiotherapy for breast cancer all are true cancer breast indicates:
EXCEPT:
a- Inoperability.
a- Has been clearly shown to improve overall survival. b- Hormonal treatment is indicated.
b- Has been clearly shown to reduce local recurrence c- N1M1 in TNM.
rates following conservative breast surgery. d- None of the above.
c- Should not be applied to the axilla, if axillary clearance
is perf ormed 87- The commonest cause of bleeding per nipple is:
d- Is the treatment of choice for the palliation of painful
bone metastases. a- Trauma to the nipple. b- Duct papilloma.
c- Mammary duct ectasia. d- Traumatic fat necrosis.
77- Adjuvant chemotherapy for breast cancer:

a- Single-agent chemotheraphy has a significantly


greater beneficial effect on survival than combination
chemotherapy.
b- Is the treatment of choice for metastases.
c- Has little role in the treatment of inflammatory cancers.
d- Has a clinical response limited to premenopausal
women

23 EDITED BY: MOHAMED ELSHEBL


ENDOCRINE 11- Which of the following is NOT an effect of
glucocorticoids ?

1- A 40-year-old woman presents with weight loss, (A) Deceased muscle protein synthesis
palpitations, and exopthalmos. On physical examination, (B) Decreased lipolysis
the thyroid gland is diffusely enlarged. Blood tests reveal (C) Inhibition of bone formation
primary hyperthyroidism. Which one of the following is (D) Increased cardiac output
not the treatment of hyperthyroidism?
12- Propouracil:
(A) Methimazoli (B) I131
(C) Subtotal thyroidectomy (D) Steroids (A) Can be given once a day in patients with
hyperthyroidism
2- Which one of the following is not part of the (B) Can cause agranulocytosis
management of a patient with hyperparathyroidism (C) Does not affect the peripheral conversion of T4 to T3
(D) Does not cross the placenta
(A) Hydration with intravenous normal saline
(B) Steroids 13- Which of The following is NoT commonly seen in
(C) Exploration of the neck for parathyroidectomy patients with MEN-I syndrome?
(D) Parathyroid scan
(A) Gastrinoma (B) Insulinoma
3- The most cmmon position of the right recurrent (C) Prolactinoma (D) Pheochromocytoma
laryngeal nerve is :
14- Painful subacute thyroiditis:
(A) Anterior to the inferior thyroid artery
(B) Posterior to the inferior thyroid artery (A) Results in hypothyroidism in >80% of patients
(C) Between the branches of inferior thyroid artery (B) Occurs most commonly in women >70 years of age
(D) Absent (non recurrent laryngeal nerve) (C) Is often preceded by upper respiratory tract infection
(D) Requires thyroidectomy for relief of symptoms in
4- The inferior parathyroid glands are derived from the: >50% of patients

(A) 1st branchial pouch (B) 2nd branchial pouch 15- The most common cause of Cushing›s syndrome is :
(C) 3rd branchial pouch (D) 4th branchial pouch
(A) Adrenal adenoma (B) Adrenal hyperplasia
5- Parathormone secretion is stimulated by: (C) Ectopic ACTH production (D) Pituitary adenoma

(A) Hyper magnesemia (B) Hypovitaminosis D 16- Surgery is indicated in which of the
(C) Parasympathetic stimulation (D) Sever hypokalemia following asymptomatic patients with primary
hyperparathyroidism?
6- Which of the following is an effect of thyroid
hormones? (A) Mildly elevated urinary calcium excretion (>100mg|dl)
(B) Reduction in creatinine clearance by 10%
(A) Positive inotropic effect on the heart (C) Serum calcium >0.8 above the upper limits of normal
(B) Maintenance of the normal hypoxic drive to breath (D) Age <50 years
(C) Increased protein turnover
(D) All of the above 17- A patient with a 1-cm medullary carcinoma of the
right thyroid and no clinically significant denoathy is
7- Secreted parathormone has a half-life of: best treated by :

(A) 2-4 minutes (B) 45-60 minutes (A) Right thyroid lobectomy and isthmusectomy
(C) 3 hours (D) 8 hours (B) Right thyroid lobectomy and subtotal left
thyroidectomy
8- Which of the following is the substrate for all (C) Total thyroidectomy
catecholamines? (D) Total thyroidectomy with central lymph node
dissection
(A) Alanine (B) Leucine
(C) Tryptophane (D) Tyrosine 18- The most common cause of primary
hyperparathyroidism is :
9- The adrenal cortex arises from:
(A) Parathyroid adenoma
(A) Ectoderm (B) Mesoderm (B) Multiple parathyroid adenomas
(C) Endoderm (D) Neural crest (C) Parathyroid hyperplasia
(D) Parathyroid carcinoma
10- The inferior adrenal artery originates from :
19- Papillary cancer of the thyroid:
(A) Phrenic artery (B) Splenic artery
(C) Renal artery (D) Aorta (A) Is uncommon in children
(B) Is the most common thyroid cancer in patients with
history of external radiation
(C) Occurs more commonly in men
(D) All of the above

SURGERY Q-BANK VOL. 1 24


20- There are 7 compartments of lymph nodes in the (C) Size > 6 cm
neck. Metastasea from thyroid cancer are uncommon in: (D) Lesion enhancement

(A) Level I nodes (B) Level III nodes 30- The most sensitive test to diagnose a
(C) Level V nodes (D) Level VII nodes pheochromocytoma is :

21- A patient with hypertension is diagnosed with (A) Plasma vanillaylmandellic acid (VMA)
hyperaldosteronism. A CT scan shows bilaterally (B) Urinary vanillaylmandellic acid (VMA)
enlarged adrenals without a mass. The most appropriate (C) Plasma metanehrines
next intervension is: (D) Urinary metanephrines

(A) Unilateral adrenalectomy 31- Which of the following cancers does NOT occur in
(B) Bilateral adrenalectomy thyroglossal duct cysts ?
(C) Selective venous catheterization
(D) Medical management (A) Papillary thyrod cancer
(B) Follicular thyrod cancer
22- The initial treatment of choice for Reidel»s thyroiditis (C) Medullary thyrod cancer
is: (D) Hurthle cell cancer

(A) Observatin (B) Antibiotics 32- Which of the following is Incorrect regarding the
(C) Corticosteroids (D) Surgery thyroid:

23- Which of the following should be the first a- The gland develops from the third pharyngeal pouch.
drug to be started in a patient with asymptomatic b- The C-cells are developed from the ultimo-branchial
pheochromocytoma ? body.
c- The weight of normal gland is 20-25 g.
(A) ACE inhibitor (B) Alpha blocker d- The resting follicle contains colloid in which iodine is
(C) Beta blocker (D) Calcium channel blocker stored.

24- Following total thyroidectomy for differentiated 33- In median ectopic thyroid which of the following
thyroid cancer, radioactive iodine ablation would be statements is True:
offered for all of the following patients EXCEPT :
a- Forms a swelling in the upper part of the neck.
(A) Stage III disease b- Usually mistaken as thyroglossal cyst.
(B) Stage II disease <45 years of age c- May be the only normal thyroid tissue present.
(C) Stage I disease with multifocal disease d- All of the above.
(D) Stage I disease <45 years of age
34- Which of the following statements regarding lingual
25- Thyridectomy should be recommended for patients thyroid is True:
with Graves› disease who :
a- This forms a rounded swelling at the foramen caecum.
(A) Are of male gender b- It may represent the only thyroid tissue.
(B) Are >55 years of age c- It may cause impairment of speech, respiratory
(C) Have large asymptomatic goiter obstruction or haemorrhage.
(D) Have a suspicious thyroid nodule d- All of the above.

26- Thyroglossal duct cysts are most commonly located: 35- Which .of the following statements regarding
thyroglossal cyst is True:
(A) On the anterior border of sternomastoid muscle
(B) In the midline at the level of the thyroid a- This may be present in any part of the thyroglossal tract.
(C) Over the medial clavicular head b- Such a cyst occupies the midline EXCEPT in the region
(D) In the midline just superior to the thyroid gland of the thyroid cartilage where the thyroglossal tract is
pushed to one side.
27- The most common cause of hyperthyroidism is : c- The swelling moves upwards on protrusion of the
tongue as well as on swallowing.
(A) Graves› disease (B) Toxic multinodular goiter d- All of the above.
(C) Plummer's disease (D) Thyroiditis
36- Which of the following statements regarding
28- The most common adrenal mass incidentally found thyroglossal fistula is False:
on CT scan is :
a- It is congenital.
(A) Adrenal cyst (B) Adrenal hemorrhage b- Follows infection.
(C) Cortical adenoma (D) Myelolipoma c- Follows inadequate removal of a thyroglossal Cyst.
d- Long standing fistulas are inclined to be situated low
29- A patient is noted to have an adrenal mass on down in the neck.
CT scan. Whch of the following CT findings is most
suggestive of adrenal cancer ? 37- Synthesis and liberation of thyroid hormone from
thyroid is controlled by:
(A) Tumor heterogencity
(B) Adjacent lymphadenopathy a- Hypothalamus. b- Hippocampus.
c- Anterior pituitary. d- Posterior pituitary.

25 EDITED BY: MOHAMED ELSHEBL


38- The distinct steps of T3 and T4 synthesis are the 47- Which of the following statements regarding isotope
following EXCEPT: thyroid scan is true

a- Trapping of inorganic iodide from the blood. a- Cold nodules are categorized as under-active nodules.
b- Oxidation of iodide to iodine. b- Warm nodules are categorized as active nodules.
c- Binding of iodine with tyrosine to form iodotyrosines. c- Hot nodules are categorized as overactive nodules.
d- No hormone remains free in the serum. d- All of the above.

39- The correct sequence of events for the metabolism of 48- For the diagnosis of which of the following condition,
iodine and synthesis of thyroid hormone is: thyroid scan is most useful:

a- Trapping, organification, coupling, release, oxidation. a- Goitre. b- Toxic adenoma of thyroid.


b- Oxidation, trapping, coupling, organification, release. c- Cysts of thyroid. . d- Thyroid malignancies.
c- Coupling, organification .. trapping, oxidation, release.
d- Trapping, oxidation, organification, coupling, release. 49- Which of the following statements regarding FNAC
in discrete thyroid swellings is True:
40- The thyroid stimulating antibody is:
a- Has been established as the investigation of choice in
a- IgG. b- IgM. c- IgA. d- IgD. discrete thyroid swelling.
b- Has got excellent patient compliance.
41- Which of the following statements regarding thyroid c- Cannot differentiate between follicular adenoma and
function test is True: follicular carcinoma.
d- All of the above.
a- T3 estimation is conclusive.
b- T4 estimation is conclusive. 50- The indication of surgery in isolated thyroid swelling
c- Serum TSH level is conclusive. is:
d- No single test is conclusive.
a- Toxic adenoma. b- Pressure symptoms.
42- Simple goiter commonly develops as a result of : c- Neoplasia. d- All of the above.
.
a- Stimulation of thyroid gland by TSH-as a result of 51- Which of the following statements regarding
inappropriate secretion from a micro-adenoma in anterior retrosternal goitre is Untrue:
pituitary.
b- Stimulation of thyroid gland by TSH to chronically low a- Often symptomless and is discovered on a routine
level of circulating thyroid hormone. chest radiograph.
c- The most important factor is dietary deficiency of b- Many of these patients attend chest clinic with
iodine. diagnosis of asthma.
d- Defective hormone synthesis probably accounts for c- My present as dysphagia. ,
many sporadic goiter. d- Recurrent laryngeal nerve paralysis is very common.

43- All of the following are goitrogens EXCEPT: 52- The symptoms of hyperthyroidism are the following
EXCEPT:
a- Vegetables of brassica family.
b- Para-amino salicylic (PAS) and calcium. a- Emotional liability. b- Heat intolerance.
c- Thyroxine. c- Weight gain. d- Excessive appetite.
d- Thiocyanate and perchlorates.
53- Which of the following statements regarding
44- Which of following statements regarding diffuse hyperthyroidism is Untrue:
hyperplasic goiter is untrue:
a- Tachycardia is common,
a- The goiter appears in childhood. b- Palm›s become hot and moist.
b- If TSH stimulation ceases the goiter may regress. c- Exophthalmos is prominent,
c- Goiter is soft, diffuse and may become large enough to d- Eight times common in-males than females.
cause discomfort.
d- The T3 level is increased. 54- True regarding Von Graefe›S sign:

45- Which of the following statements regarding a- Lagging behind of the upper eyelid.
multinodular goitre is True: b- Retraction of the upper eyelid with infrequent
wrinkling,
a- Nodules are usually multiple forming a multinodular c- Absence of wrinking of the forehead.
goitre. d- Convergence of the eyes is difficult.
b- This is one form of a clinically solitary nodule.
c- Nodule may be colloid or cellular and cystic 55- In paediatric Graves› disease, surgery is preferred
degeneration and haemorrhage are common. because:
d- All of the above.
a- Radioiodine is potentially carcinogenic.
46- All the following are complications in nodular goiter b- Antithyroid drugs are not without side effects.
EXCEPT: c- Radioiodine causes late myxoedema
d- All of the above.
a. Tracheal obstruction. b. Secondary thyrotoxicosis.
c. Carcinoma. d. Enophthalmos.

SURGERY Q-BANK VOL. 1 26


56- Regarding treatment of thyrotoxicosis during A. It presents clinically as a solitary nodule.
pregnancy all are true Except: B. In adenoma there is no invasion of the capsule or of
peri-capsular blood vessels.
a- treatment of thyrotoxicosis during pregnancy is C. Preferable treatment is lobectomy
predominantly with antithyroid drugs. D. All of the above.
b- Anti-thyroid drugs should be used with the highest
possible doses. 65- Most common primary malignant thyroid tumor is:
c- Propylthiouracil (PTU) is the drug of choice,
predominantly because of its protein binding which leads a- Papillary. b- Follicular.
to less transplacental transfer. c- Anaplastic. d- Medullary.
d- Radioiodine treatment is contraindicated during
pregnancy and lactation. 66- Which of the following is not correct in the
presentation of carcinoma of the thyroid:
57- The following can be used to block the production of
thyroid hormone EXCEPT: a- Earache.
b- Hoarseness of the voice.
a. Propylthiouracil b. Propranolol c- A pulsating bone tumor. .
c. Methimazole d. Carbimazole d- The sex ratio is 3 males to 1 female.

58- The disadvantage of antithyroid drugs in the 67- All of the following are True about papillary
treatment of thyrotoxicosis is: carcinoma of the thyroid EXCEPT:

a- Treatment is prolonged. a- It is the slowest growing malignant tumor of the


b- Failure rate after 2 years is 50 %. thyroid.
c- It is impossible to predict which patient will go into b- It may change into the anaplastic variety.
remission. c- It metastasizes primarily by hematogenous spread.
d- All of the above. d- It is dependent on thyroid-stimulating hormone (TSH)
stimulation.
59- The advantages of surgery in thyrotoxicosis is:
68- Advantages of total thyroidectomy for management
a- the goiter is removed. of papillary carcinomas of the thyroid, include all the
b- The cure is rapid. following EXCEPT:
c- The cure rate is high if surgery is adequate.
d- All of the above. a- Possibility of using radioactive iodine postoperatively to
identify and treat metastases.
60- Which of the following treatment schedule for b- The ability to use thyroglobulin levels as a marker for
diffuse toxic goitre is True: recurrence.
c- Lower overall recurrence rate.
a- Over 45years: radioactive iodine. d- Lower risk of hypoparathyroidism.
b- Under 4S years: with large goitre-surgery.
c- Under 45 years: with small goitre-antithyroid drugs. 69- The term lateral aberrant thyroid implies:
d- All of the above.
a- Congenital aberrant thyroid tissue lateral to the thyroid.
61- Surgery is the treatment of choice in toxic nodular b- A metastasis in a cervical lymph node from an occult
goitre because of the following EXCEPT: thyroid carcinoma.
c- A metastasis from carcinoma of the larynx.
a- Does not respond to antithyroid drugs rapidly. d- A type of branchial cyst.
b- b- Does not respond to radioiodine rapidly.
c- Goitre itself is often large and uncomfortable. 70- In which of the following carcinoma of thyroid
d- None of the above. ‹psammoma bodies› are seen:

62- Which of the following statements regarding tension a- Follicular. b- Medullary.


haematoma deep to cervical fascia following subtotal c- Papillary. d- Anaplastic
thyroidectomy is True:
71- Thyroid cancer with multiple bone metastasis is most
a- Slippage of a ligature on the superior thyroid artery. likely to be of which type:
b- Occasionally hemorrhage from a thyroid remnant.
c- Bleeding thyroid veins. A. Medullary. B. Papillary.
d- All of the above. C. Anaplastic. D. Follicular.

63- Which of the following statement regarding 72- Which of the following statements regarding
thyrotoxic crisis (storm) is untrue: medullary carcinoma is Untrue:

a- Is an acute exacerbation of hyperthyroidism. a- Tumors of C-cells derived from neural crest.


b- If a thyrotoxic patient is inadequately prepared for b- Tumors of cells of thyroid follicle.
thyroidectomy. c- There is a characteristic amyloid stroma.
c- May follow an unrelated operation. d- High levels of serum calcitonin are produced.
d- None of the above.

64- Which of the following statement regarding


follicular adenoma is True:

27 EDITED BY: MOHAMED ELSHEBL


73- In thyroid carcinoma, mediastinal node involvement a- Are normally two in number
is a feature of which type: b- Lie inside the capsule of the thyroid.
c- Are reddish-blue in color.
a- Follicular. b- Anaplastic. d- Are not the secretors of calcitonin.
c- Papillary. d- Medullary.
83- Which of the following clinical features is not related
74- Diarrhea in medullary carcinoma of thyroid is due to: to hyperparathyroidism:

a- Serotonin. b- Calcitonin. a- Risus sardonicus. b- Psychic moans.


c- Thyroxine. d- Hypercalcemia. c- Abdominal groans. d- Pain from renal stones.

75- Which of the following is not a part of multiple 84- In primary hyperparathyroidism choose the correct
endocrine neoplasia Type IIa? answer:

a- Medullary thyroid carcinoma a- Result from single or multiple adenomas.


b- Pheochromocytoma. b- Associated with multiple bone cysts.
c- Parathyroid adenoma. c- Can present with renal stones.
d- None of the above. d- All are true.

76- Regarding medullary carcinoma of the thyroid all are 85- Chvostek›s sign is:
true EXCEPT:
a- Carpal spasm induced by sphygmomanometer cuff
a. Is a tumor of the parafollicular C cells pressure on the upper arm above the systolic blood
b. Produce thyroxin as the principle hormone pressure for not more than two minutes in the normal
c. The majority of cases are sporadic person.
d. Can occur as part of the MEN type 1I syndrome b- As above, but the person has tetanus.
c- Twitching of the facial muscles produced by tapping
77- All of the following statements regarding De- over the branches of the facial nerve in front of the tragus
Quervain›s thyroiditis is True EXCEPT: of the ear in the normal person
d- As above, but the person has tetany.
a. This is due to virus infection.
b. ESR is raised. 86- About APUD cells, all the following statements are
c. Thyroid antibodies are absent. true, except:
d. Iodine 123 uptake is high.
A. These are of ectodermal origin arising from the neural
78- The classical presentation of Hashimoto›s thyroiditis crest.
is: B. Neoplasms of APUD cells may occur simultaneously in
different organs.
a. Presents as a multinodular 95oiter. C. APUD cell neoplasms are always malignant.
b. Thyroid failure is common. D. APUD cells are present in the islets of Langerhans.
c. Primary myxoedema without detectable thyroid
enlargement represents the end stage of pathological 87- About multiple endocrine neoplasia (MEN)
process. syndromes, all the following statements are true, except:
d. All of the above.
A. The syndromes are non-hereditary.
79- Ideal treatment of Hashimoto›s thyroiditis is: B. Neoplasms may be benign or malignant.
C. Islet cell tumours of the pancreas are part of MEN I.
a- Subtotal thyroidectomy. b- Steroids. D. Medullary carcinoma of the thyroid is a part of MEN II.
c- Thyroxin. d- TSH
88- About anatomy of the thyroid gland, one statement
80- False regarding Hashimoto›s thyroiditis is: only is true:

a- Autoimmune thyroiditis. A. The thyroid is enclosed within the prevertebral fascia.


b- Women commonly affected. B. The sternomastoid muscles lie anterior to the thyroid.
c- Hvpothyroid state. C. The thyroid gland extends up to the level of the hyoid
d- Always requires thyroidectomy. bone.
D. Lymphatic drainage of the thyroid gland goes primarily
81- Ligation of the superior thyroid artery above the to the submandibular lymph nodes.
superior pole of the thyroid gland carries with it the risk
of: 89- About anatomical relations of the thyroid gland, all
the following statements are true, except:
a- Injury to the recurrent nerve.
b- Injury to the vagus nerve. . A. The external laryngeal nerve is related to the superior
c- Devascularizarion of the inferior parathyroid gland. thyroid vessels.
d- Injury to the external branch of the superior laryngeal B. The recurrent laryngeal nerve is related to the inferior
nerve. thyroid artery.
C. The recurrent laryngeal nerve lies in the tracheo-
82- Regarding the parathyroid glands, they: oesophageal groove.
D. The thyroid isthmus lies at the level of the cricothyroid
membrane.

SURGERY Q-BANK VOL. 1 28


90- About the blood vessels of the thyroid gland, all the 98- About treatment of Graves› disease all the following
following statements are true except: statements are true, except:

A. The superior thyroid artery is a branch of the external A. Thyroidectomy is the first-line treatment.
carotid artery. B. Agranulocytosis is a possible complication of anti-
B. The inferior thyroid artery is a branch of the common thyroid drugs.
carotid artery. C. Radioactive iodine therapy is contraindicated during
C. The superior thyroid vein drains into the internal jugular pregnancy.
vein. D. Radioactive iodine therapy is better avoided in cases of
D. The middle thyroid vein drains into the internal jugular exopthalmos.
vein.
99- About toxic multinodular goitre, all the following
91- All the following statements about thyroiditis are statements are true. except:
true, except:
A. It usually affects patients at an older age than those
A. Subacute thyroiditis is caused by viral infection. with Graves› disease.
B. Subacute thyroiditis predisposes to thyroid lymphoma. B. Exopthalmos is more prominent than in patients with
C. Hashimoto›s thyroiditis is an autoimmune disease. Graves› disease.
D. Hashimoto›s thyroiditis ultimately ends in C. Heart failure is more likely to occur than in patients
hypothyroidism. with Graves› disease.
D. Goitre is usually sizable.
92- About the aetiology of simple goitre, all the following
statements are true, except: 100- About stridor in the postoperative period after
thyroidectomy, all the following statements are true,
A. Pendred›s syndrome is characterized by except:
hypothyroidism, goitre and blindness.
B.Pendred›s syndrome is caused by deficiency of A. Unilateral recurrent laryngeal nerve injury causes
peroxidase enzyme. stridor.
C. Deficiency of iodine causes simple goitre. B. Dyspnoea due to deep neck haematoma should be
D. Pregnancy raises the need for iodine. immediately evacuated.
C. Laryngeal oedema is a cause of stridor.
93- A 17-year old female presents with a 3cm thyroid D. Tracheal collapse due to tracheomalacia is a rare cause
nodule. FNA shows papillary structures. There is no of stridor.
evidence of metastases. The appropriate treatment is:
101- About anatomy of the parathyroid glands, all the
A. Thyroxine therapy. B. Radioactive iodine therapy. following statements are true, except:
C. Excision of the nodule. D. Total thyroidectomy.
A. There are always four parathyroid glands.
94- The clinical manifestations of thyroid malignancy B. The superior parathyroids are derived from the fourth
include all the following, except: branchial pouches while the inferior glands are derived
from the third pouches.
A. Neck pain. C. The blood supply of the parathyroid glands is mainly
B. Progressive enlargement of a neck mass. from the inferior thyroid arteries.
C. Hoarseness of voice. D. The inferior parathyroid glands may be present in the
D. Jaundice. mediastinum.

95- One of the following cancers may be multicentric: 102- About primary hyperparathyroidism, one statement
only is true:
A. Papillary carcinoma of the thyroid.
B. Follicular carcinoma of the thyroid. A. Commonest pathology is parathyroid hyperplasia.
C. Anaplastic carcinoma of the thyroid. B. Serum calcium is reduced.
D. Medullary carcinoma of the thyroid. C. Serum calcitonin is reduced.
D. Osteoporosis is one of its consequences.
96- About papillary thyroid cancer, all the following are
true, except: 103- The manifestations of primary hyperparathyroidism
include all the following, except:
A. It is the commonest thyroid cancer.
B. It usually affects teen-agers. A. Bony pains.
C. It usually spreads by lymphatics. B. Renal colic.
D. It secretes calcitonin. C. Epigastric pain.
D. Normal serum calcium and high PTH.
97- About the types of thyrotoxicosis, all the following
statements are true, except: 104- The differential diagnoses of hypercalcaemia
include all the following, except:
A. In Graves› disease the gland has no nodularity. .
B. Plummer›s disease is thyrotoxicosis that develops on A. Primary hyperparathyroidism.
top of a multinodular 98oiter. B. Bone metastases.
C. Graves› disease is characterized by huge enlargement C. Multiple myeloma.
of the thyroid gland. D. Tuberculosis.
D. A solitary hyperactive thyroid nodule can cause
thyrotoxicosis.

29 EDITED BY: MOHAMED ELSHEBL


105- About the investigations for clinically-suspected A. Hypertension. B. Diabetes mellitus.
hyperparathyroidism, all the following statements are C. Delayed wound healing. D. Bronchospasm.
true, except:
113- Retrosternal goiter is detected by:
A. High serum calcium is present in patients with primary
hyperparathyroidism. a- The lower border of the thyroid is always seen and felt.
B. Estimating PTH is necessary for the diagnosis of b- Resonance behind the sternum.
hyperparathyroidism. c- Positional dyspnea.
C. Estimating serum calcitonin is necessary for diagnosis. d- None of the above.
D. Ultrasound scan is used for localization of the adenoma.
114- The following are applied to papillary carcinoma of
106- About post-thyroidectomy hypoparathyroidism, all the thyroid except:
the following statements are true, except:
a- Is almost always unifocal.
A. It may be caused by inadvertent removal of the b- Histologically displays Psammoma bodies.
parathyroid glands at thyroidectomy. c- Typically spread to the cervical lymph nodes.
B. It may be caused by devascularization of the d- May require total thyroidectomy.
parathyroid glands at operation.
C. The condition usually presents about two weeks after 115- The following are applied to solitary thyroid nodules
thyroidectomy. except:
D. The earliest symptom is facial and peripheral
numbness. a- More than 20 % of them are malignant in adult
population.
107- About anatomy of the adrenal glands, all the b- Less than 20 % of them are malignant.
following statements are true, except: c- May be associated with secondary hyperthyroidism.
d- Surgical removal is the treatment of choice in most
A. The right adrenal gland is closely related to the inferior cases.
vena cava.
B. The right adrenal vein ends in the inferior vena cava. 116- In a thyroglossal fistula all are correct except:
C. The left adrenal vein ends in the left renal vein.
D. The left adrenal vein is shorter than the right vein. a- Has the internal opening beside the fraenum lingulae.
b- Follows inadequate removal of a thyroglossal cyst.
108- About physiology of the adrenal gland, all the c- Externally has a hood of skin with concavity upwards.
following statements are true, except: d- Occurs in carcinoma of the tongue.

A. Aldosterone is secreted by the zona glomerulosa of the


adrenal cortex.
B. Aldosterone secretion is under control of the rennin-
angiotensin system. VASCULAR SURGERY
C. A high serum cortisol level inhibits the secretion of
ACTH from the anterior pituitary.
D. The adrenal medulla is under control by the anterior
pituitary gland. 1- A 65-year-old female on her routine examination was
noted to have a pulsatile abdominal mass. She has been
109- A 57 years old man is accidentally discovered to otherwise healthy with history of hypertension with
have a well-defined 2.5cm right adrenal mass on CT scan no other history, except family history of father dying
that is done for some other reason. Hormonal studies are of ruptured abdominal aortic aneurysm. What are the
all normal. The proper management is: acceptable reasons to operate on abdominal aortic
aneurysms in 65-year-old female with5-cm infrarenal
A. FNAC. B. Adrenalectomy. aneurysm?
C. Radiological follow-up. D. Radiotherapy.
(A) Presence of aneurysm
110- What statement is true of phaeochromocytoma? (B) Aneurysm with intramural thrombus
(C) Asymptomatic aneurysm 5.5 cm
A. It is a neuroendocrine tumour of the adrenal cortex. (D) Associated 2-cm iliac aneurysm
B. It is associated with M EN I.
C. It is malignant in 90% of cases, 2- An 89-year-old male presents with asymptomatic
D. Extra-adrenal pheochromocytomas account for about 8-cm abdominal aneurysm. He has a recent history of
10% of cases. myocardial infarction (MI) and is not a candidate for
coronary artery bypass. What should the treatment
111- For the pre and intraoperative preparation of a options include?
phaeochromocytoma patient, all the following are used,
except: (A) Conservative treatment observation
(B) Computerized axial tomography (CAT) scan to
A. Alpha adrenergic blockers. evaluate eligibility for endovascular repair
B. Beta adrenergic blockers. (C) Open repair without any further workup
C. Correction of deficient blood volume. (D) Axillofemoral bypass and coil embolization of
D. Correction of hypokalaemia. aneurysm

112- The risks of adrenalectomy for Cushing›s syndrome


are increased by all the following, except:

SURGERY Q-BANK VOL. 1 30


3- A70-year-old male underwent an open abdominal 9- A 57-year-old male smoker is referred to you because
aortic aneurysm repair for ruptured aneurysm. He of two episodes of right upper extremity weakness
was stable during the procedure. In intensive care over the past 6 months, each lasting for 10–15 minutes.
unit he was noted to have no urine output and was Findings on CT scan of the head are negative. An
also noted to have large bloody bowel movement on angiogram shows a75% stenosis of the left carotid
first postoperative day. The next step for investigation artery. What is the most appropriate treatment?
includes:
(A) Antiplatelet therapy
(A) Reexploration (B) Oral anticoagulants
(B) Arterial blood gas evaluation for acidosis (C) Carotid endarterectomy
(C) Sigmoidscopy/colonoscopy (D) Surgery only if a stroke develops
(D) Antibiotics and hydration
10- A 24-year-old man complains of progressive
4- A 69-year-old man was noted to have abdominal pain intermittent claudication of the left leg. On examination,
in left flank with severe hypotension and pulsatile mass the popliteal, dorsalis pedis, and posterior tibial pulses
in abdomen. He was taken to the operating room are normal; but they disappear on dorsiflexion of the
after he coded in the emergency room. Which of the foot. What is the most likely diagnosis?
following statements regarding ruptured abdominal
aortic aneurysm is TRUE? (A) Embolic occlusion
(B) Atherosclerosis obliterans
(A) Mortality is about 10%. (C) Popliteal artery entrapment syndrome
(B) Aortic control is usually obtained by thoracotomy. (D) Cystic degeneration of the popliteal artery
(C) It cannot be treated by endovascular means.
(D) Mortality following a code for ruptured AAA is 100% 11- A 45-year-old woman undergoes cardiac
catheterization through a right femoral approach. Two
5- A82-year-old female presented with history of loss of months later, she complains of right lower extremity
vision in right eye for about 15 minutes and it cleared up. swelling and notes the appearance of multiple
She has a history of diabetes and hypertension. She had varicosities. On examination, a bruit is heard over the
which showed old infarct on right side. Carotid duplex right groin. What is the most likely diagnosis?
showed that patient had 99% carotid artery stenosis.
Which one of the following statements is TRUE? (A) Femoral artery thrombosis
(B) Superficial venous insufficiency
(A) 60% chance that extra cranial carotid artery stenosis (C) Arteriovenous (AV) fistula
is the cause of transient ischemic attack (TIA). (D) Deep vein insufficiency
(B) It is always due to platelet emboli.
(C) 25% may be intracranial bleed. 12- A young basketball player develops an acute onset
(D) It is always due to thrombosis. of subclavian vein thrombosis (effort thrombosis) after
heavy exercise. What is the next step in management?
6- A60-year-old male patient with bilateral carotid artery
stenosis 90%, with history of right-sided weakness with (A) Active exercise of the limb
resolution of symptoms in 15 minutes. How would you (B) Anti-inflammatory drugs
treat the patient? (C) Thrombolytic therapy
(D) First-rib resection
(A) Right carotid endarterectomy
(B) Left carotid endarterectomy 13- A 40-year-old patient undergoes a CT scan of the
(C) Start patient on aspirin abdomen for nonspecific abdominal pain. A splenic
(D) Start patient on heparin artery aneurysm is incidentally identified. What is true of
the splenic artery aneurysm?
7- A 72-year-old patient is noted to have neurological
deficit following elective carotid endarterectomy in (A) It requires splenectomy for optimal treatment.
recovery room. What is the most appropriate treatment (B) It is more common in men.
at this time? (C) It is caused by atherosclerosis in most cases
(D) It may rupture during pregnancy.
(A) Carotid duplex
(B) CAT scan of brain 14- A70-year-old man with a long-standing history of
(C) Angiogram of cerebral vessels diabetes develops gangrene of the right second toe.
(D) Exploration of the same side What is true of his diabetic foot?

8- A 40-year-old chronic smoker presents with ulceration (A) Dorsalis pedis and posterior tibial arteries are always
of the tip of the right second, third, and fourth toes. absent.
He gives a history of recurrent migratory superficial (B) Gangrene of the toe always requires urgent below-
phlebitis of the feet occurring a few years ago. Physical knee amputation.
examination findings are remarkable for absent bilateral (C) His right femoral artery is most probably occluded or
posterior tibial and dorsalis pedis pulses with palpable stenosed.
popliteal pulses. What is the single most important step (D) Trophic ulcers are sharply demarcated.
in management?

(A) Multiple toe amputations


(B) Long-term anticoagulant therapy
(C) Immediate operative intervention
(D) Cessation of smoking

31 EDITED BY: MOHAMED ELSHEBL


15- Eleven years after undergoing right modified radical (A) An atherosclerotic plaque
mastectomy, a 61 -year-old woman develops raised red (B) An abdominal aortic aneurysm
and purple nodules over the right arm. What is the most (C) Heart (D) Paradoxical embolus
likely diagnosis?
23- An elderly patient with ischemic rest pain is found to
(A) Lymphangitis (B) Lymphedema have combined aortoiliac and femoropopliteal occlusive
(C) Lymphangiosarcoma (D) Hyperkeratosis disease. What is the treatment of choice?

16- A middle-age woman has right leg and foot (A) Aortofemoral bypass (B) Femoropopliteal bypass
nonpitting edema associated with dermatitis and (C) Aortofemoral and femoropopliteal bypass
hyperpigmentation. The diagnosis of chronic venous (D) Vasodilator therapy
insufficiency is made. What is the treatment of choice?
24- A 66-year-old woman has a 5.5-cm infrarenal
(A) Vein stripping (B) Pressure-gradient stockings abdominal aortic aneurysm. What is the most common
(C) Skin grafting (D) Perforator vein ligation manifestation of such an aneurysm?

17- A 55-year-old woman has bilateral leg edema (A) Abdominal or back pain
associated with thick, darkly pigmented skin. A (B) Acute leak or rupture
Trendelenburg’s test is done, and results are interpreted (C) Incidental finding on abdominal examination
as positive/positive. What does this patient have? (D) Atheroembolism

(A) Competent varicose veins/competent perforators 25- A young woman develops a left femoral
(B) Competent varicose veins/incompetent perforators arteriovenous fistula a few months after a stab wound to
(C) Incompetent varicose veins/competent perforators the groin. Which of the following physiological changes
(D) Incompetent varicose veins/incompetent perforators (Nicoladoni-Branham sign) is elicited on physical
examination?
18- A 70-year-old executive is complaining of three-
block intermittent claudication of both legs. What is the (A) Appearance of CHF when the artery proximal to the
percentage chance of his developing limb-threatening fistula is compressed
gangrene? (B) Slowing of the pulse rate when the fistula is
compressed
(A) Less than 10% (B) 20% (C) A rise in the pulse rate when the artery distal to the
(C) 60% (D) More than 75% fistula is compressed
(D) A bruit heard only after the fistula is occluded
19- Thirty-six hours after undergoing an abdominal
aortic aneurysm repair, a 70- year-old woman develops 26- A young patient sustains blunt trauma to his right
abdominal distension associated with bloody diarrhea. knee that results in acute thrombosis of his popliteal
What is the most likely diagnosis? artery. Which tissue is most sensitive to ischemia?

(A) Aortoduodenal fistulas (A) Muscle (B) Nerve


(B) Diverticulitis (C) Skin (D) Fat
(C) Pseudomembranous enterocolitis
(D) Ischemic colitis 27- Ahomeless elderly man is brought to the emergency
department after sustaining frostbite to both feet. What
20- An 18-year-old man develops a painful, swollen is the most appropriate immediate management?
leg while training for the New York Marathon. There is
tenderness in the calf and ecchymosis is present. What is (A) Slow rewarming at room temperature
the most likely diagnosis? (B) Amputation of the gangrenous toes
(C) Rapid rewarming with warm water
(A) Cellulitis (D) Rapid rewarming with hot water or dry heat
(B) DVT
(C) Superficial thrombophlebitis 28- A middle-aged man complains of shortdistance
(D) Tear of the plantaris muscle claudication in the right thigh. The angiogram shows a
right common iliac artery stenosis of 90% over a short
21- Four days after suffering MI, a 78-year-old woman segment. What is the treatment of choice?
suddenly develops severe diffuse abdominal pain. Her
electrocardiogram (ECG) shows atrial fibrillation. On (A) Aortofemoral bypass
examination, the abdomen is soft, minimally tender, (B) Left-to-right fermorofemoral bypass
and slightly distended. Hyperactive bowel sounds are (C) Iliofemoral bypass
present. What is the most likely diagnosis? (D) PTA and stent placement

(A) Mesenteric embolus 29- A 65-year-old man with hypertension and a blood
(B) Nonocclusive ischemic disease pressure of 190/105 mm Hg has unilateral renal artery
(C) Perforated peptic ulcer stenosis. What is the best diagnostic test to determine
(D) Congestive heart failure (CHF) the physiologic significance of the lesion?

22- A 60-year-old man with a history of atrial fibrillation (A) Aortography (B) Renal ultrasound
is found to have a cyanotic, cold right lower extremity. (C) Renal vein renin assay
The embolus is most probably originating from which of (D) Rapid-sequence intravenous pyelogram
the following?

SURGERY Q-BANK VOL. 1 32


30- An elderly patient complains of recurrent 38- All of the following statements concerning popliteal
episodes of amaurosis fugax. This is attributable to artery aneurysms are true EXCEPT
microembolization of which of the following?
a- Approximately 50% arc associated- with aneurysms at
(A) Facial artery (B) Retinal artery other sites
(C) Posterior auricular artery b- Rupture into the popliteal space is a frequent
(D) Superficial temporal artery complication
c- Associated thrombosis carries a high risk of amputation
31- A 65-year-old woman television technician d- Associated distal embolization may result in tissue loss
undergoes femoral embolectomy and leg fasciotomy.
Following surgery, she is noted to have oliguria, and her 39- A diabetic patient presented with advanced
urine is red. What is the most probable diagnosis? ischemic gangrene of the foot and rest pain. On
examination, the pedal pulses were absent but the
(A) Hematuria secondary to heparin popliteal pulse was felt. Angiography showed occlusion
(B) Embolus of the renal artery of the tibial arteries with no distal run- off. The classical
(C) Myoglobinuria (D) Hemoglobinuria treatment for this patient is:

32- A newborn girl with family history of lymphedema is a- Popliteal-to-distal bypass b- Syme's amputation
noted to have bilateral lower extremity swelling. What is c- Below knee amputation d- Above knee amputation
the diagnosis?
40- In lower limb acute embolic ischemia, the embolus
(A) Lymphedema praecox (B) Milroy disease may originate from all of the following sites EXCEPT:
(C) Lymphedema tarda (D) Meigs’s syndrome
a- The heart over recent myocardial infarction
33- Claudication is: b- The deep veins from extensive lower limb DVT
c- The heart with mitral valve disease and atrial fibrillation
a- Pain at rest. b- Pain relieved by rest. d- The abdominal aorta with aortic aneurysm
c- Constant pain. d- Pain not relieved by rest.
41- Diabetic patients are more prone to develop foot
34- Rest pain refers to pain: ulcers. All the following are important contributing
causes EXCEPT:
a- Anywhere in the body at rest.
b- In the thigh of the patient with Buerger›s disease. a- Diabetic patients usually have peripheral neuropathy
c- In the back affecting their feet
d- In the foot of a patient of severe vascular disease. b- Diabetic foot deformities renders the foot more
susceptible to trauma
35- By definition, which of the following patients is c- Diabetic patients have exaggerated inflammatory
having critical limb ischemia: response to infection
d- Diabetic patients may have concomitant chronic
a- A patient presenting with acute limb ischemia and ischemia
impending gangrene
b- A patient presenting with a chronic ischemic foot 42- If a diabetic patient presents with gangrene of one of
ulcer and rest pain his toes, the first thing that should be done is to:
c- A patient presenting with calf claudication progressing
over the past 3 months a- Amputate the gangrenous toe to prevent spread of
d- A patient presenting with infective gangrene of his toe gangrene
and intact pedal pulse. b- Ask for fasting blood sugar to control blood sugar level
c- Ask for serum lipid profile to correct possible
36- All the following statements describe ischemic rest hyperlipidcmia
pain EXCEPT: d- Look for pedal pulses to evaluate foot circulation

a- Continuous severe aching, or burning pain that 43- Once you diagnose acute lower limb ischemia, the
becomes worth at night first thing to do is:
b- Sudden cramping pain in the calf that awakens the
patient from sleep. a- Angiography to differentiate between embolic and
c- Partially relieved by putting the leg below the level of thrombotic ischemia
the heart b- Catheter directed embolectomy under local anesthesia
d- Increases if there is superadded infection in the c- Give the patient heparin to avoid clot propagation
ischemic foot d- Transfer the patient to a vascular surgery center

37- One of the following statements describes an 44- One of the following is NOT a component of» Leriche
ischemic ulcer: Syndrome» (infrarenal aorto-iliac occlusion):

a- Superficial painful ulcer above the ankle surrounded a- Bilateral absent femoral pulse
with pigmented eczematous skin b- Bilateral feet parathesia
b- Deep painless ulcer in the sole reaching down to the c- Buttock & thigh claudication
bone with intact pedal pulse d- Impotence
c- Superficial painful ulcer on the heal of a diabetic
patient with absent pedal pulse
d- Deep painless ulcer between the toes of 11 diabetic
patient with gangrenous floor and intact pedal pulse

33 EDITED BY: MOHAMED ELSHEBL


45- In acute ischemia, catheter directed thrombolysis 55- In acute embolic occlusion, heparin is given to:
can be used in all of the following conditions EXCEPT:
a- Dissolve embolus.
a- Recent acute thrombosis of less than 3 days duration b- Reduce extension.
b- Viable limbs with lax muscles and intact sensations c- Maintain patency of distal vessels.
c- Patient with history of major surgery one and a half d- All of the above.
months ago
d- Patient with history of cerebral stroke one and half 56- The most important prognostic sign of acute
months ago ischaemia of a limb is:

46- All the following can result in acute limb ischemia, a- Pallor. b- Cold skin.
EXCEPT: c- Cutaneous anaethesia. d- Muscle turgor.

a- Embolism originating from the heart with chronic atrial 57- Indicate the Incorrect statement about arterial
fibrillation embolism:
b- Acute hemolysis of RBCs in a patient with known
spherocytosis a- It results in acute ischaemia.
c- Thrombosis of a diseased artery on top of chronic lower b- Is always due to a detectable site of thrombosis.
limb ischemic c- Is often due to lodgement of an embolism at the
d- Traumatic fracture of bones with injury to nearby bifurcation of a main artery.
arteries d- Is associated with a much higher incidence of
gangrene than simple ligation of the same artery.
47- In acute ischemia, one of the following is a sign of
irreversible ischemia that will need amputation: 58- Intra-arterial thrombolysis is best achieved with:

a- Marked delay in the capillary refilling time a- Streptokinase. b- Urokinase.


b- Marked swelling & turgidity of calf muscles c- tPA infusion.
c- Loss of foot superficial and deep sensation d- Pulse-spray tPA (tissue plasminogen activator).
d- Paralysis of the small muscles of the foot
59- In fat embolism, the fat is most likely to arise from:
48- Reconstructive arterial surgery is recommended for
patients with the following manifestations of ischemia a- Bone marrow. b- Adipose tissue.
EXCEPT: c- Chylomicrons. d- None of the above.

a- Ischemic neuropathy. b- Trophic ulceration. 60- Therapeutic embolisation is used for:


c- Toe gangrene. d- Claudication.
a- Arrest of haemorrhage from GI &urinary tract.
49- Which reconstructive surgery has good long-term b- Treatment of a-v malformation.
outcome: c- Shrinkage oftumour growth.
d- All of the above.
a- Aortoiliac. b- Femoropopliteal.
c- Femorodistal. d- All of the above. 61- Moist gangrene occurs in presence of:

50- The graft used in aortoiliac bypass is: a- Venous and arterial occlusion. b- Embolism.
c- Diabetes. d- All of the above.
a- Long saphenous vein graft b- Dacron
c- PTFE d- Umbilical vein 62- Peripheral aneurysm most commonly involves:

51- The most satisfactory graft material for a- Femoral artery. b- Axillary artery.
femoropoplitcal bypass grafting is: c- Popliteal artery. d- Brachial artery.

a- Autogenous vein graft b- Woven dacron. 63- Lumbar sympathectomy is of value in the
c- Knitted dacron. d- Gortex. management of:

52- In arterial by-pass surgery the best vein to use is: a- Intermittent claudication b- A-V fistula
c- Diabetic neuropathy
a- Cephalic vein b- Femoral vein d- Distal ischemia affecting skin of toes
c- Long saphenous vein d- Short saphenous vein
64- Sympathectomy is not advised for:
53- The MOST common site at which arterial emboli
lodge is the: a- Raynaud›s disease of the fingers.
b- Hyperhidrosis of the feet.
a- Aortic bifurcation. c- Intermittent claudication. d- Acrocyanosis.
b- Common iliac bifurcation.
c- Common femoral bifurcation. 65- An overdose of Heparin is treated by:
d- Cerebral circulation.
a- Prostaglandins. b- Phenidione.
54- Fogarty catheter is used for: c- Protamine sulphate. d- Prostigmine.

a- IV nutrition b- Ureteric catheterization


c- Arteriography d- Arterial embolectomy

SURGERY Q-BANK VOL. 1 34


66- In Buerger's disease, the following statements are 75- In lumbar sympathectomy which of the following
correct EXCEPT: ganglia is spared:

a- Phlebitis migrans may occur. a- L4 b- L3 c- L1 d- L2


b- Intermittent claudication is the most common
symptom. 76- Where are the Cockett perforators present?
c- Rest pain is a grave symptom.
d- Only the lower limbs are affected. A. Lateral thigh. B. Medial lower leg.
C. Lateral lower leg. D. Femoral triangle.
67- In Buerger's disease, the following statements are
true EXCEPT: 77- Thrombolysis is achieved by:

a- Severe pain never occurs. A. Decreased activity of thrombin.


b- Recurrent exacerbations and remissions are B. Converting thrombin to fibrin.
characteristic. C. Converting plasminogen to plasmin.
c- Intermittent claudication is usually the first symptom. D. All of the above.
d- May end in gangrene.
78- Inherited predisposition to deep vein thrombosis
68- Which statement is untrue concerning Raynaud›s includes all the following causes, except:
disease:
A. Factor V Leiden mutation (heterozygous and
a- Occurs equally in both sexes. homozygous)
b- Usually manifests itsel f in the third decade. B. Antithrombin deficiency.
c- Affects the lingers and hands and rarely the feet. C. Protein 5 excess.
d- Is characterized by recurrent episodes initiated by cold D. Protein C deficiency.
or emotional stress.
79- All the following factors increase the susceptibility to
69- Concerning abdominal aortic aneurysms, which of postoperative deep vein thrombosis (DVT) except:
the following statements is untrue:
A. Anaemia. B. Obesity.
a- Are commonest in the sixth and seventh decades. C. The intake of contraceptive pills.
b- Have a much higher incidence in males than females. D. Malignancy.
c- Characteristically originate below the renal arteries.
d- Never extend beyond the aortic bifurcation. 80- Which of the following veins are valveless?

70- The cardinal signs and symptoms of sudden A. Great saphenous vein. B. Short saphenous vein.
peripheral ischemia include: C. Portal vein. D. Common femoral vein.

a- Pain. b- Paresthesia 81- Which of the following should be monitored in


c- Paralysis d- All of the above patients who are anticoagulated with heparin?

71- Cause of A-V fistula: A.Bleeding time. B. PT.


C.APTT. D. None of the above.
a- Congenital b- Blunt trauma
c- Penetrating trauma d- All of the above 82- Which of the following is monitored during
anticoagulation with Warfarin?
72- The haemodynamic effects of an arteriovenous
fistula in the thigh include the following Except: A. Platelet count. B. Bleeding time.
C.INR. D. APTT.
a- Decreased peripheral resistance.
b- Reduced cardiac output. 83- What is the time needed to achieve adequate
c- Increased venous pressure. anticoagulation in patients who are starting Warfarin?
d- Reduced diastolic pressure.
A.6 hours. B. 24 hours.
73- Branham's sign refers to: C.48 hours. D.72 hours.

a- Bruit heard over an arteriovenous fistula. 84- About peri-operative anticoagulant prophylaxis, all
b- The increased growth of the limb involved with the the following are true, except:
arterio-venous fistula a child.
c- The occurrence of heart failure where there is a large A. Should be started immediately after surgery.
arterio-venous fistula present. B. Is indicated for obese patients.
d- A sharp decrease in the pulse rate when the arterio- C. Heparin is the usual agent.
venous fistula is close- by digital pressure. D. Prophylaxis is continued for one week after surgery.

74- The Adson maneuver for thoracic outlet syndrome is 85- What is the initial dose of heparin in the patients
positive with: with DVT?

a- Numbness and tingling in the hand. A.45 units/kg. B. 60 units/kg.


b- Pallor of the hand. C.80 units/kg. D.None of the above.
c- Disappearance of radial pulse.
d- Acrocyanosis in the hand.

35 EDITED BY: MOHAMED ELSHEBL


86- What is the preferred method of treatment of a- Occurs only in the legs.
ulceration due to chronic venous insufficiency? b- Are dilated lengthened veins.
c- Are results of valvular incompetence
A. Compression therapy (elastic stocking). d- May follow venous thrombosis.
B. Saphenous vein ligation and stripping.
C.Ligation of perforating veins (Linton operation). 96- All the following are venous symptoms EXCEPT:
D.None of the above.
a- Aching. b- Leg cramps.
87- Which of the following is an indication for inferior c- Palpitation. d- Tiredness.
vena cava (IVC) filter insertion?
97- The best operation for varicose vein is:
A. Recurrent DVT despite adequate anticoagulation.
B. Contraindication of anticoagulation in proximal DVT a- Multiple subcutaneous ligations
with a suspected one episode of PE. b- Injecting sclerosants throughout.
C. Pulmonary showering in patients with DVT and c- Sub-fascial ligatures.
moderate pulmonary hypertension. d- Division or ligation at sites of communication from
D. All of the above. deep to superficial system.

88- In the first episode of DVT following a major 98- The Brodie-Trendelenberg test is used to detect:
operation, how long should the patient be given
warfarin? a- The presence of deep femoral vein thrombosis.
b- The integrity of the long saphenous vein .
A.4 weeks. B. 6 weeks. c- The presence of an incompetent valve at the
C.3-6 months. D.One year. saphenofemoral junction.
d- The presence of valves in the inferior vena cava.
89- The laboratory test that is used for diagnosis of DVT
and/or pulmonary embolism is: 99- A patient is receiving anticoagulation for treatment
of deep venous thrombosis. Which of the following
A. D-dimer. B. ESR. findings suggest that he is given warfarin
C.Platelet count. D.Prothrombin time.
a- Anticoagulation is monitored by measuring the
90- The imaging study of choice for the diagnosis of prothrombin time
suspected deep vein thrombosis is: b- Anticoagulation can be reversed by giving protamine
sulfate
A. Ascending venography. B. Duplex scan. c- Anticoagulation is achieved within one hour of drug
C. Isotope scan. D. CT angiography. administration
d- The anticoagulant is administered by subcutaneous
91- About stripping of the long saphenous vein, all the injection
following statements are true, except:
100- A patient with primary varicose veins and normal
A. It is not an essential part of operations for primary deep system can present by any of the following
varicose veins. EXCEPT:
B. It can cause injury of the saphenous vein.
C. It can cause injury of the sural nerve. a- Discomfort on prolonged standing
D. It deprives the patient of a vein that could be used for b- Lower limb muscle cramps
coronary bypass. c- Severe lower limb edema d- Venous ulcer

92- The term venous pump refers to: 101- A patient with dilated tortuous veins over his lower
abdomen filling predominantly from below upwards.
a- A part of autotransfusion apparatus. This clinical finding can be associated with:
b- The left atrium.
c- The apparatus for rapid transfusion of blood. a- Chronic Superior vena-cava obstruction
d- Musculofascial anatomy and physiology of calf. b- Chronic inferior vena-cava obstruction
c- Chronic portal hypertension
93- Pressure in superficial veins in leg during standing is: d- Chronic pelvic congestion

a- 20 mmHg. b-40mmHg. 102- In a patient with chronic venous insufficiency due to


c- 80 mmHg. d- 120 mmHg. residual deep venous obstruction, which of the following
describes the clinical picture of his limb?
94- What happens to superficial venous pressure during
exercise: a- Soft pitting edema with pigmentation and eczema
b- Hard pitting edema without pigmentation and eczema
a- Rises. b- Falls. c- Soft pitting edema without pigmentation and eczema
c- Unchanged. d- None of the above. d- Hard pitting edema with pigmentation and eczema

95- Regarding varicose veins all statements are true


EXCEPT:

SURGERY Q-BANK VOL. 1 36


103- Which of the following statements describes a- Sapheno- femoral disconnection (Trendlenberg
infected venous ulcer? operation)
b- Stripping of the long saphenous in the thigh only
a- Painless superficial ulcer with skin pigmentation c- Stripping of the long saphenous in the leg and thigh
around it d- Elastic compression stocking
b- Painful superficial ulcer with absent peripheral
pulsations 110- In acute deep venous thrombosis, the
c- Painless deep ulcer with absent peripheral pulsations recommended treatment protocol is:
d- Painful superficial ulcer with skin pigmentation
around it. a- Low molecular weight heparin for the whole duration
of treatment
104- Severe prolonged venous congestion of the lower b- Initial heparin anticoagulation followed by warfarins
limbs may result in an acute inflammatory condition c- Oral anticoagulants for the whole duration of treatment
known as acute lipodermatosclerosis. The main line of d- Initial heparin anticoagulation followed by low dose
treatment in this condition is: salicylates.

a- Anti-inflammatory drugs and external compression 111- Thrombolytic therapy is effective in all of the
b- Broad spectrum antibiotic and external compression following patients Except:
c- Anti-inflammatory drugs and broad spectrum
antibiotic a- Chronic ilio-femoral deep venous thrombosis with
d- Venotonic drugs and broad spectrum antibiotic severe edema
b- Massive pulmonary embolism with acute pulmonary
105- Which of the following best describes a healing hypertension
ulcer? c- Acute deep venous thrombosis with impending venous
gangrene
a- Small ulcer with undermined edge and red granulating d- Acute arterial thrombosis with a viable non critical limb
floor ischemia
b- Large ulcer with sloping edge and red granulating floor
c- Small ulcer with raged edge and bleeding floor 112- Spontaneous superficial thrombophlebitis occurs in:
d- Large ulcer with raised edge and necrotic floor
a- Polycythemia. b- Polyarteritis.
106- One of the complications of 1ry varicose veins is c- Buerger's disease. d- All of the above.
acute superficial thrombophlebitis. The main line of
treatment for this condition is: 113- The appropriate management of thrombophlebitis
of superficial veins is:
a- Systemic anti-inflammatory drugs and external
compression a- Supportive bandages and ambulation.
b- Local anti-inflammatory ointments and external b- Supportive bandages and strict bed rest.
compression c- Anticoagulants and bed rest.
c- Systemic broad spectrum antibiotic and external d- Anticoagulants and ambulation.
compression
d- Local antibiotic ointments and external compression 114- High risk patient for DVT include:

107- The main aim of investigations in a patient with a- Neoplasia. b- Pelvic surgery.
chronic venous insufficiency is to: c- Congestive heart failure. d- All of the above

a- Identify coagulation defects and hyper coagulable 115- Post-operative deep venous thrombosis is suspected
states from the following EXCEPT:
b- dentify major systemic problems that may alter the
operative decision a- Unexplained post-operative fever or tachycardia.
c- Identify venous reflux and confirm deep venous b- Pain in the sole or calf.
patency c- Swelling or oedema of the calf or leg.
d- Identify congenital arterio-venous fistula to avoid d- Pain on plantar flexion of the foot.
operating on them
116- Phlegmasia albs dolens or white leg is due to:
108- The accurate dynamic investigation used to
evaluate superficial and deep venous reflux is: a- Lymphatic obstruction.
b- Femoral deep vein thrombosis.
a- Pocket Doppler c- Thrombosis of IVC and lymphatic obstruction
b- Ascending venography d- Iliofemoral vein thrombosis plus lymphatic
c- Duplex ultrasound obstruction.
d- Descending venography
117- Phlegmasia cerulae dolens or painful purple swelling
109- A 32 year old healthy male presented with is characterized by all the following EXCEPT:
symptomatic varicosity of the long saphenous vein.
Duplex ultrasound showed severe saphenofemoral a- Extensive iliofemoral thrombosis
reflux normal deep venous system and competent leg b- Propagation of the thrombosis proximally and distally
perforators. What treatment would you advise? c- Reduced arterial inflow
d- Treated by anticoagulants

37 EDITED BY: MOHAMED ELSHEBL


118- The treatment of phlegmasia cerulae dolens is: A. Internal carotid artery (ICA).
B. External carotid artery (ECA).
a-Nearly always surgical.(i.e., venous thrombectomy, C. Middle meningeal artery.
catheter-directed lytic therapy, or both) D. None of the above.
b- Anticoagulation.
c- Low molecular weight dextran. 128- Diagnosis of renal artery hypertension is best
d- Supportive. achieved by:

119- The commonest cause of fatal pulmonary embolism A. Renal artery duplex ultrasound.
is: B. Selective angiography.
C.MRA
a- Iliac vein thrombosis. D.None of the above.
b- Thrombophlepitis of the femoral vein.
c- Calf vein thrombosis. 129- Which is true regarding endovascular abdominal
d- Axillary vein thrombosis. aortic aneurysm repair?

120- Pulmonary embolism is best diagnosed by: A. It costs the same as open repair:
B. Requires lifelong follow-up imaging.
a- Chest x-ray. b- Spiral CT. C. Results in longer hospital stay than open repair.
c- MRI. d- Doppler. D. Requires more blood transfusion than open repair.

121- The following are clinical features of a large 130- About revascularization procedures, all the
pulmonary embolism EXCEPT: following statements are true, except:

a- Pleuritic chest pain b- Haemoptysis A. The patency rate of aortobifemoral bypass is lower
c- low grade fever d- Collapsed neck veins than that of femoro-popliteal bypass.
B. Short-segment iliac artery stenosis can be successfully
122- In patients with a proven pulmonary embolus all are treated by percutaneous angioplasty.
true EXCEPT: C. A synthetic graft is used for aortobifemoral bypass.
D. Reversed saphenous vein is used for femoropopliteal
a- Only 10% have clinical evidence of deep venous bypass.
thrombosis
b- The classic ECG features are described S I Q3T3 131- What is the initial treatment in patients with acute
c- Arterial blood gases are abnormal in all patients lower limb ischemia?
d- Low molecular weight heparins are as effective a
heparin infusion A. Thrombolysis. B. Surgical embolectomy.
C.Heparinization. D. None of the above.
123- What is the normal ankle brachial index (ABI)?
132- The following statements about abdominal aortic
A. More than 1. B. Less than 0.8 aneurysms (AAA) are all true, except:
C. 0.9. D. 1.
A. The commonest cause is atherosclerosis
124- Which artery supplies collateral flow between the B. They are usually present above the level of renal
celiac artery (CA) and superior mesenteric artery (SMA)? arteries
C. The main risk is rupture
A. Middle colic artery. D. Mural thrombi can migrate to cause distal embolization
B. Pancreaticoduodenal arteries.
C. Left colic artery. 133- About abdominal aortic aneurysm (AAA), all the
D. Marginal artery of Drummond. following statements are true, except:

125- Which arteries supply collateral flow between the A. Normal diameter of the abdominal aorta is 2-3cm.
superior mesenteric artery (SMA) and inferior mesenteric B. Aortic aneurysms with a diameter of 5cm are advised
artery (IMA)? to have urgent repair.
C. The main risk of abdominal aortic aneurysm is rupture.
A. Marginal artery of Drummond. D. One of its complications is distal embolization.
B. Inferior pancreaticoduodenal artery.
C. Left gastric artery. 134- About Buerger›s disease, all the following
D. Splenic artery. statements are true, except:

126- What is the best conduit to be used in A. It mostly affects the iliac arteries.
femoropopliteal bypass operations? B. It affects male only.
C. It affects smokers only.
A. Human umbilical vein. B. Gore-Tex PTFE. D. Pain is an early symptom.
C.Dacron. D.Autogenous vein.
135- All the following are strong signs of arterial injury,
127- Occlusion of which of the following arteries causes except:
amaurosis fugax?
A. Any haematoma in the course of an artery.
B. External arterial bleeding.
C. Absent distal pulses.
D. A palpable thrill at the site of trauma.

SURGERY Q-BANK VOL. 1 38


136- The commonest source of arterial lower limb emboli 2- Among the following, which is the most common type
is: of basal cell carcinoma (BCC)?

A. Thrombus in thoracic aortic aneurysm. A. The superficial spreading type.


B. Thrombus in abdominal aortic aneurysm. B. The pigmented type.
C. Cardiac mural thrombus on top of myocardial C. The nodular type.
infarction. D. None of the above.
D. Left atrial thrombus.
3- Which type of melanoma has the best 5 year survival?
137- About arterial embolism, all the following
statements are true, except: A. Nodular type.
B. Superficial spreading type.
A. The commonest are thrombemboli. C. Lentigomaligna.
B. A throl1lembolus usually impacts at the bifurcation of D. Acral lentiginous.
an artery.
C. Embolectomy can be done under local anaesthesia. 4- What is the safety margin at the time of excision of a
D. Tense calf muscles are a bad prognostic sign. 2.5 mm thick melanoma?

138- Virchow's triad is: A.1cm. B. 2 cm. C.3cm. D.5 cm.

a- Three malignant lymph nodes enlargement in the left 5- About depth of a burn, all the following statements
supraclavicular fossa. are true, except:
b- Three malignant lymph nodes enlargement in the right
supraclavicular fossa. A. Burns that are caused by hot water (scalds) tend to be
c- Venous blood stasis, venous endothelial injury and of partial thickness.
hypercoagulability. B. Partial-thickness burns tend to be less painful than
d- Obstructed jaundice, porta hepatis lymphadenopathy full-thickness ones.
and septicemia. C. If kept clean and protected, partial-thickness burns heal
within three weeks.
139- Which of the following doesn`t apply to 1ry varicose D. Partial-thickness burns show blisters.
veins of lower limbs:
6- About burn eschar, all the following statements are
a- May lead to disfigurement of lower limbs. true, except:
b- Intermittent claudication is a prominent feature in
their presentations. A. A circumferential limb eschar may cause ischaemia of
c- Associated with sapheno-femoral incompetence in this limb.
most cases. B. A circumferential chest eschar may cause respiratory
d- One must test for patency of deep system before doing impairment.
surgery. C. Eschar is insensitive tissue that can be divided without
anaesthesia.
140- The term «venous pump» refers to: D. Eschar is formed of stiff fibrous tissue.

a- The apparatus used for rapid blood transfusion. 7- About inhalation burns, all the following statements
b- The left atrium. are true, except:
c- The presence of valves in the inferior vena cava.
d- The musclofascial anatomy and physiology of the calf. A. They are likely to happen with flame burns in confined
places.
141- The saphenous nerve is liable to be injured during B. They should be suspected with facial burns.
long saphenous vein stripping: C. Airway obstruction is the main threat to life.
D. All inhalation-burn victims should have tracheostomy.
a- Behind the knee. b- In the groin.
c- In the leg. d- In the mid thigh. 8- All the following are indications for admission of a
burn victim to hospital, except:

A. A 10% flame burn that affects the face of an adult.


B. A 13% scald of the trunk in a 4-year old boy.
C. A 10% burn that involves the perineum.
PLASTIC SURGERY D. A 10% burn that affects the front of the abdomen.

9- Improvement in burn survival is attributed to:

A. Prophylactic antibiotics.
1- Capillary haemangioma of the face requires which of B. Nutritional support (enteral and parenteral).
the following investigation before starting treatment? C. Early excision and grafting.
D. Fresh frozen plasma administration.
A. MRI of the sinuses.
B. CT of the brain.
C. Liver ultrasound.
D. Venous Duplex of the lower limbs.

39 EDITED BY: MOHAMED ELSHEBL


10- All the following statements about sebaceous cysts A. Radial aspect of the wrist.
are true, except: B. Dorsal aspect of the wrist.
C. Volar aspect of the wrist.
A. They are common in children. D. Ulnar aspect of the wrist.
B. They do not appear on the palm.
C. They are attached to the skin. 21- What is the most common tumour of the hand?
D. Sometimes the punctum cannot be seen.
A. Fibroma. B. Angioma.
11- Precancerous skin lesions include all the following, C. Enchondroma. D. Osteoclastoma.
except;
22- Which of the following statements for squamous cell
A. Bowen›s disease . carcinoma is Untrue:
B. xerederrnta pigmentosa
C. Giant hairy nevus in children a- A malignant tumour arising from epidermis.
D. Keratoacanthoma (molluscum sebaceurn) b- Less rapidly growing tumour than basal cell carcinoma.
c- Frequently occurs in pre-existing skin lesion.
12- A giant hairy nevus is excised mainly because of: d- Environmental carcinogen is the cause.

A. The risk of malignant transformation 23- A port wine stain ( Naevus flammeus ) is:
B. The unsightly appearance
C. The frequent bleeding on contact with clothing a- A premalignant lesion of the skin.
D. The frequent attacks of infection b- A type of melanoma.
c- A type of bruising of the skin.
13- A graft taken at a dermal level including elements of d- A type of haemangioma.
epidermis and dermis;
24- Which of the following cells or blood elements play a
A. Is a full thickness graft. role in the initial phases of wound healing?
B. Requires 10 days before blood flow is established in it.
C. Is used when large areas of skin are required. a- Polymorphonuclear leukocytes (PMNs)
D. All of the above. b- Macrophages.
c- Lymphocytes d- All the above.
14- Dupuytren›s contracture is best treated by:
25- The following statements regarding the role of
A. Surgery. B. Steroids. macrophages in the wound healing process are true
C. Splinting. D. Physiotherapy. EXCEPT:

15- Phalen›s and reversed phalen›s test are diagnostic in: a. Macrophagcs are the dominant cell type during the
inflammatory phase of wound healing.
A. Carpal tunnel syndrome. B. Neuroma. b. Macrophages are essential for wound healing
C. Malunion of the scaphoid bone. c. The macrophage role in wound healing is limited to
D. Trigger finger. phagocytosis
d.Macrophages are a source of a number of humoral
16- Ganglion cyst is most commonly located at: factors essential for wound healing

A. Anterior aspect of the wrist. 26- Concerning the remodeling phase of wound healing
B. Posterior aspect of the wrist. choose the correct answer:
C. Dorsum of the hand.
D. Palmar surface of the hand. a. Total collagen content increases steadily through this
phase.
17- What is the most common hand anomaly? b. The normal adult ratio of collagen is approximately 4: 1
of type I to type III collagen is restored.
A. Syndactyly. B. Club hand. c. Eventually a scar will achieve the strength of
C. Polydactyly. D. Constriction band. unwounded skin.
d. The normal adult ratio of collagen is approximately 4: 1
18- Drainage of distal pulp space infection is done of type III to type I collagen is restored.
through:
27- Healing by first intention means:
A. Bilateral incision of each side of the finger.
B. Incise the nail fold. a. Using catgut.
C. Transverse incision on the fingertip. b. Union between two edges of an incision without
D. Short longitudinal skin incision on area of maximum subsequent breakdown.
fluctuance not crossing the interphalangeal joint crease. c. Subsequent breakdown.
d. The immediate use of a plastic dressing
19- Stenosing tenosynovitis (trigger finger) is best treated
initially by: 28- A hypertrophic scar:

A. Early surgery. b B. Physiotherapy. a- Is the same as a keloid scar.


C. Corticosteroid injection. D. None of the above. b- Requires radiotherapy for a cure.
c- Raised red scar which persists for a year or two before
20- De Quervain Tenosynovitis presents with pain at the: becoming white.
d- Only occurs on the abdomen.

SURGERY Q-BANK VOL. 1 40


29- Keloids are characterized by the following EXCEPT: Immediate treatment should include.

a- Consist of dense overgrowth of scar tissue. a- Amputation. b- Escharotomy.


b- Develop after wounds, burns and vaccination marks. c- Application of ice. d- Arteriogram.
c- Occur most often on the face, neck and front of the
chest. 39- In Extensive burns, which of the following
d-May turn malignant. statements is incorrect concerning fluid replacement
during the first 24 hours?
30- Wolfe graft is:
a- May consist of Ringer lactate alone (parkland formula).
a- A partial thickness skin graft. b- Should be controlled by the hourly urine output.
b- A pinch skin graft. c- Is calculated according to the body weight and the
c- A small full thickness skin graft surface area,
d- A pedicle graft d- Is given at a uniform rate.

31- Split-thickness skin grafts differ from full-thickness 40- Which of the following influences wound
grafts in all of the following EXCEPT that they: dehiscence:

a- Contain part of the dermis. a-Type of incision. b- Type of closure.


b- Take even when the conditions of the recipient bed are c- Anaemia-hypoproteinemia. d- All of the above.
suboptimal.
c- Are less cosmetically acceptable. 41- Steroids are responsible for all of the following
d- Develop little pigmentation after transfer. EXCEPT:

32- Which of the following skin grafts has least a- Decreased wound tensile strength.
contraction? b- GI ulceration.
c- Gall stone formation.
a- Full thickness. b- Split thickness. d- Avascular necrosis of hip.
c- Both have equal contraction. d- All of the above.
42- Factors that decrease collagen synthesis include all
33- The aim of treatment of an infant with a cleft lip is to: of the following EXCEPT:

a- Improve appearance. a- Hpoproteinaernia b- Infection.


b- Make feeding possible. c- Anemia. d- Advanced age.
c- Achieve adequate speech.
d- Achieve adequate dentition. 43- «Sign of emptying» is seen in:

34- In carcinoma of the tongue which of the following is a- Salmon patch. b- Port wine stain.
incorrect: c- Plexi form angioma. d- Strawberry haemangioma.

a- It is virtually symptomless in its early stages. 44- In skin graft transfer the word «take» of graft refers
b- One form of presentation is a fissure. to:
c- It may cause earache.
d- The usual site is the back of the tongue. a- Healing of graft.
b- Vascularisation of grail.
35- In the rule of nine for calculation of (body surface c- Dense attachment of graft to surrounding tissue.
area) BSA, the entire back accounts for: d- Epithelial ingrowth in the margins.

a- 9 percent. b- 18 percent, 45- Regarding skin grafts choose the correct statement:
c- 24 percent. d- 36 percent.
a- Split-skin grafts contain epidermis only.
36- What is the Best method of initial airway b- Split-Skin grafts maintain their own blood supply.
management for this patient? c- Thicker split-skin grafts are more likely to ‹take› than
thinner ones.
a- Continue with 2 L nasal cannula pending blood gas d- Split-skin grafts result in greater contraction than full-
result. thickness.
b- 40% oxygen via face mask.
c- Endotracheal intubation. 46- Cleft lip should be repaired at the age of:
d- Needle cricothyroidotomy.
a. 2 years. b. 3 months.
37- At what initial rate (using the Parkland formula) c. 6-9 months. d. 9-12 months.
should intravenous lactated Ringer›s solution be
administered: 47- Skin flaps can be used to cover the following except:

a- 340 cc/hr. b- 420 cc/hr. a- Poor vascular or avascular bed.


c- 550 cc/hr. d- 630 cc/hr. b- Exposed bony areas.
c- Full thickness loss of tissues of lid and nose.
38- The patient›s right upper extremity becomes d- None of the above.
cyanotic and cool, with delayed capillary refill, loss of
pulses, and no flow signal on Doppler ultrasound exam.
The nearest burn center is 3 hours away.

41 EDITED BY: MOHAMED ELSHEBL

You might also like